Property - Case Digests

July 22, 2017 | Author: Kimberly Arriola | Category: Property, Foreclosure, Mortgage Law, Ownership, Search And Seizure
Share Embed Donate


Short Description

case digests...

Description

San Beda College of Law CIVIL LAW REVIEW  

PROPERTY CLASSIFICATION OF PROPERTY: Movable Property

  LADERA v. HODGES CA-G.R. No. 8027-R, 23 September 1952 [ESTEBAN]

FACTS: Ladera entered into a contract with Hodges whereby the latter promised to sell a lot subject to the condition that in case of failure of the purchaser to make monthly payment within 60 days after due, the contract is considered as rescinded and annulled. It is likewise stipulated that in such case, all sums of money paid would be considered rentals and the vendor shall be at liberty to dispose of the parcel of land with all the improvements theron to any other person in a manner as if the contract had never been made. After the execution of the contract, Ladera built on a lot a house of mixed materials assessed at P4500. Unfortunately, Ladera failed to pay the agreed installments, whereupon the appellant rescinded the contract and filed an action for ejectment. The MTC rendered a decision upon agreement of the parties —Ladera to vacate and surrender possession of the lot and pay P10 a month until delivery of the premises. The court issued an alias writ of execution and pursuant thereto the sheriff levied upon all rights, interests, and participation over your house standing on the lot. The sheriff posted the notices of the sale but did not publish the same in a newspaper of general circulation. At the auction sale Ladera did not attend because she had gone to Manila and the sheriff sold the property to Avelina Magno as the highest bidder. On July 6, 1948, Hodges sold the lot to Manuel Villa and on the same day the latter purchased the house from Magno for P200 but this last transaction was not recorded. Ladera returned to Iloilo after the sale and learned of its results. She went to see the sheriff and upon the latter’s representation that she could redeem the property, she paid him P230 and the sheriff issued a receipt. It does not appear, however, that this money was turned over to Hodges. Thereupon, Spouses Ladera filed an action against Hodges, the sheriff, and the judgment sale purchasers, Magno and Villa to set aside the sale and recover the house. The lower court ruled in favor of Ladera. Hodges et al contend that the house being built on land owned by another person should be regarded in law as movable or personal property. ISSUE: Should the house built on land owned by another be regarded as immovable property?

1 4S 2014-2015  

San Beda College of Law CIVIL LAW REVIEW   RULING: YES. According to Article 334 of the Civil Code (now 415), immovable property are the following: xxx “Lands, building, roads, and constructions of all kinds adhering to the soil;” xxx. Applying the principle ubi lex non distinguit nec nos distinguere debemos (when the law does not distinguish we ought not to distinguish), the law makes no distinction as to whether the owner of the land is or is not the owner of the building. In the case of immovables by destination, the Code requires that they be placed by the owner of the tenement, in order to acquire the same nature or consideration of real property. In cases of immovable by incorporation, the Code nowhere requires that the attachment or incorporation be made by the owner of the land. The only criterion is union or incorporation with the soil. A true building (not merely superimposed on the soil) is immovable or real property, whether it is erected by the owner of the land or by usufructuary or lessee. (Note: Due to the unavailability of the original text via the internet, the digest above was sourced from several digests of the original case. Rest assured that due diligence was exercised to verify everything provided herein.)

MINDANAO BUS COMPANY v. THE CITY ASSESSOR & TREASURER and the BOARD OF TAX APPEALS of Cagayan de Oro City G.R. No. L-17870, 29 September 1962 [FERNANDEZ] FACTS: Petitioner is a public utility solely engaged in transporting passengers and cargoes by motor trucks. It owns a land where it maintains and operates a garage for its TPU motor trucks; a repair shop; blacksmith and carpentry shops, and with machineries placed therein, its TPU trucks are made; body constructed; and same are repaired in a condition to be serviceable in the TPU land transportation business it operates. The machineries have never been or were never used as industrial equipment to produce finished products for sale, nor to repair machineries, parts and the like offered to the general public indiscriminately for business or commercial purposes. Respondent City Assessor of Cagayan de Oro City assessed at P4,400 petitioner's above-mentioned equipment. Petitioner appealed the assessment to the respondent Board of Tax Appeals on the ground that the same are not realty. The Board of Tax Appeals of the City sustained the city assessor, so petitioner herein filed with the Court of Tax Appeals a petition for the review of the assessment. The CTA held the petitioner liable to the payment of the realty tax on its maintenance and repair equipment mentioned above. Hence, this petition. 2 4S 2014-2015  

San Beda College of Law CIVIL LAW REVIEW   ISSUE: Should the tools and equipment in the petitioner company’s repair shop be considered immovable taxable real properties? RULING: NO. Movable equipment to be immobilized in contemplation of the law must first be "essential and principal elements" of an industry or works without which such industry or works would be "unable to function or carry on the industrial purpose for which it was established." The tools and equipment in question are not essential and principle municipal elements of petitioner's business of transporting passengers and cargoes by motor trucks. They are merely incidentals — acquired as movables and used only for expediency to facilitate and/or improve its service. The transportation business could be carried on without the repair or service shop if its rolling equipment is repaired or serviced in another shop belonging to another. MAKATI LEASING AND FINANCE CORPORATION v. WEAREVER TEXTILE MILLS, INC. G.R. No. L-58469, 16 May 1983 [MADRILENO] FACTS: Private respondent Wearever Textile Mills, Inc., discounted and assigned several receivables with Makati Leasing and Finance Corporation under a Receivable Purchase Agreement. To secure the collection of the receivables assigned, private respondent executed a Chattel Mortgage over certain raw materials inventory as well as a machinery described as an Artos Aero Dryer Stentering Range. Upon private respondent's default, petitioner filed a petition for extrajudicial foreclosure of the properties mortgage to it. However, the Deputy Sheriff assigned to implement the foreclosure failed to gain entry into private respondent's premises and was not able to effect the seizure of the aforedescribed machinery. Petitioner filed a complaint for judicial foreclosure for which the court issued a writ of seizure, the enforcement of which was however subsequently restrained upon private respondent's filing of a motion for reconsideration. On July 13, 1981, the sheriff enforcing the seizure order, repaired to the premises of private respondent and removed the main drive motor of the subject machinery. The Court of Appeals reversed the ruling of the trial court and said that the machinery in suit cannot be the subject of replevin, much less of a chattel mortgage, because it is a real property pursuant to Article 415 of the new Civil Code, the same being attached to the ground by means of bolts and the only way to remove it from respondent's plant would be to drill out or destroy the concrete floor, the reason why all that the sheriff could do to enforce the writ was to take the main drive motor of said machinery. 3 4S 2014-2015  

San Beda College of Law CIVIL LAW REVIEW   ISSUE: Is the property in question a real or personal property RULING: PERSONAL PROPERTY. It must be pointed out that the characterization of the subject machinery as chattel by the private respondent is indicative of intention and impresses upon the property the character determined by the parties. As stated in Standard Oil Co. of New York v. Jaramillo, 44 Phil. 630, it is undeniable that the parties to a contract may by agreement treat as personal property that which by nature would be real property, as long as no interest of third parties would be prejudiced thereby. If a house of strong materials, like what was involved in the case of Tumalad v. Vicencio, may be considered as personal property for purposes of executing a chattel mortgage thereon as long as the parties to the contract so agree and no innocent third party will be prejudiced thereby, there is absolutely no reason why a machinery, which is movable in its nature and becomes immobilized only by destination or purpose, may not be likewise treated as such. This is really because one who has so agreed is estopped from denying the existence of the chattel mortgage. SANTOS EVANGELISTA V. ALTO SURETY & INSURANCE CO. INC. G.R. No. L-11139, 23 April 1958 [MAGULTA] FACTS: On June 4, 1949, Santos Evangelista instituted a civil case (entitled Santos Evangelista v. Ricardo Rivera) for a sum of money against Ricardo Rivera. On the same day, he obtained a writ of attachment, which levied upon a house built by Rivera on a land leased to him. In due course, judgment was rendered in favor of Evangelista who bought the house at public auction held in compliance with the writ of execution. The corresponding definite deed of sale was issued to him on on October 22, 1952, upon expiration of the period of redemption. When Evangelista sought to take possession of the house, Rivera refused to surrender it upon the ground that he leased the property from the Alto Surety & Insurance Co., Inc. and that the latter is now the true owner of said property. After due trial, the Court of First Instance of Manila rendered judgment in favor of Evangelista. The decision was reversed by the CA upon the ground that although the writ of attachment in favor of Evangelista had been filed with the Register of Deeds prior to the sale in favor of respondent, Evangelista did not acquire thereby a preferential lien, the attachment having been levied as if the house in questIon were immovable property, although in the opinion of the CA, it is “ostensibly a personal property.” As such, the Court of Appeals held, "the order of attachment . . . should have been served in the manner provided in subsection (e) of section 7 of Rule 59," of the Rules of Court ISSUE: Should a house built on a leased property by the lessee be considered as movable or personal property for the purpose of attachment? 4 4S 2014-2015  

San Beda College of Law CIVIL LAW REVIEW   RULING: NO. The house is not personal property, much less a debt, credit or other personal property not capable of manual delivery, but immovable property. As explicitly held, in Laddera v. Hodges, “a true building (not merely superimposed on the soil) is immovable or real property, whether it is erected by the owner of the land or by usufructury or lessee. It is true that the parties to a deed of chattel mortgage may agree to consider a house as personal property for purposes of said contract. However, this view is good only insofar as the contracting parties are concerned. It is based, partly, upon the principle of estoppel. Neither this principle, nor said view, is applicable to strangers to said contract. Much less is it in point where there has been no contract whatsoever, with respect to the status of the house involved, as in the case at bar. TSAI v. HON. COURT OF APPEALS G.R. No. 120098, 2 October 2001 [MANRIQUE] FACTS: Respondent Ever Textile Mills, Inc. (EVERTEX) obtained a loan from petitioner Philippine Bank of Communications (PBCom). As security for the loan, EVERTEX executed in favor of PBCom, a deed of Real and Chattel Mortgage over the lot under TCT No. 372097, where its factory stands, and the chattels located therein as enumerated in a schedule attached to the mortgage contract. On April 23, 1979, PBCom granted a second loan to EVERTEX. The loan was secured by a Chattel Mortgage over personal properties enumerated in a list attached thereto. These listed properties were similar to those listed in the first mortgage deed. Afterwards, EVERTEX purchased various machines and equipments. On November 19, 1982, due to business reverses, EVERTEX filed for insolvency. All its assets were taken into the custody of the Insolvency Court, including the collateral, real and personal, securing the two mortgages. In the meantime, PBCom, commenced extrajudicial foreclosure proceedings against EVERTEX. On March 7, 1984, PBCom consolidated its ownership over the lot and all the properties in it. It leased the entire factory premises to petitioner Ruby L. Tsai and sold the factory, lock, stock and barrel to her, including the contested machineries. EVERTEX filed a complaint for annulment of sale, reconveyance, and damages against PBCom, alleging inter alia that the extrajudicial foreclosure of subject mortgage was not valid, and that PBCom, without any legal or factual basis, appropriated the contested properties which were not included in the Real and Chattel Mortgage of the first mortgage contract nor in the second contract which is a Chattel Mortgage, and neither were those properties included in the Notice of Sheriff's Sale.

5 4S 2014-2015  

San Beda College of Law CIVIL LAW REVIEW   ISSUE: Are the controverted parties movable properties? RULING: The intent of the parties must be looked at. While it is true that the controverted properties appear to be immobile, a perusal of the contract of Real and Chattel Mortgage executed by the parties herein gives a contrary indication. In the case at bar, both the trial and the appellate courts reached the same finding that the true intention of PBCOM and the owner, EVERTEX, is to treat machinery and equipment as chattels. As ruled by the appellate court: As stressed upon by appellees, appellant bank treated the machineries as chattels; never as real properties. Indeed, the 1975 mortgage contract, which was actually real and chattel mortgage, militates against appellants' posture. It should be noted that the printed form used by appellant bank was mainly for real estate mortgages. But reflective of the true intention of appellant PBCOM and appellee EVERTEX was the typing in capital letters, immediately following the printed caption of mortgage, of the phrase "real and chattel." So also, the "machineries and equipment" in the printed form of the bank had to be inserted in the blank space of the printed contract and connected with the word "building" by typewritten slash marks. Now, then, if the machineries in question were contemplated to be included in the real estate mortgage, there would have been no necessity to ink a chattel mortgage specifically mentioning as part III of Schedule A a listing of the machineries covered thereby. It would have sufficed to list them as immovables in the Deed of Real Estate Mortgage of the land and building involved. As regards the 1979 contract, the intention of the parties is clear and beyond question. It refers solely to chattels. The inventory list of the mortgaged properties is an itemization of sixty-three (63) individually described machineries while the schedule listed only machines and 2,996,880.50 worth of finished cotton fabrics and natural cotton fabrics. SERG'S PRODUCTS, INC v. PCI LEASING AND FINANCE G.R. No. 137705, 22 August 2000 [MATEO] FACTS: Respondent PCI Leasing and Finance, Inc, filed with the RTC-QC a complaint for a sum of money with an application for a writ of replevin. Respondent Judge issued a writ of replevin directing its sheriff to seize and deliver the machineries and equipment to PCI after 5 days and upon the payment of the necessary expenses. In the implementation of the said writ, the sheriff proceeded to petitioner’s factory, seized one machinery with word that he would return for the other. Petitioner asserted that properties sought to be seized were immovable as defined in Article 415 of the Civil Code. The motion was opposed by PCI Leasing, on the ground that the properties were still personal and therefore 6 4S 2014-2015  

San Beda College of Law CIVIL LAW REVIEW   still subject to seizure and a writ of replevin. In its decision on the original action for certiorari filed by the Petitioner, the appellate court, Citing the Agreement of the parties, held that the subject machines were personal property, and that they had only been leased, not owned, by petitioners; and ruled that the "words of the contract are clear and leave no doubt upon the true intention of the contracting parties." ISSUE: Did the machineries become real property by virtue of immobilization? RULING: NO. The machines are personal property and they are proper subjects of the Writ of Replevin. In the present case, the machines that were the subjects of the Writ of Seizure were placed by petitioners in the factory built on their own land. They were essential and principal elements of their chocolate-making industry. Hence, although each of them was movable or personal property on its own, all of them have become “immobilized by destination because they are essential and principal elements in the industry.” However, contracting parties may validly stipulate that a real property be considered as personal. The Agreement between the parties provides “The PROPERTY is, and shall at all times be and remain, personal property notwithstanding that the PROPERTY or any part thereof may now be, or hereafter become, in any manner affixed or attached to or embedded in, or permanently resting upon, real property or any building thereon, or attached in any manner to what is permanent.” After agreeing to such stipulation, they are consequently estopped from claiming otherwise. Under the principle of estoppel, a party to a contract is ordinarily precluded from denying the truth of any material fact found therein. It should be stressed, however, that the Court’s holding-that the machines should be deemed personal property pursuant to the Lease Agreement-is good only insofar as the contracting parties are concerned. Hence, while the parties are bound by the Agreement, third persons acting in good faith are not affected by its stipulation characterizing the subject machinery as personal. BURGOS, SR. v. CHIEF OF STAFF G.R. No. L-64261, 26 December 1984 [MEDINA] FACTS: Assailed in this petition for certiorari prohibition and mandamus with preliminary mandatory and prohibitory injunction is the validity of two [2] search warrants issued by respondent Judge Ernani Cruz-Pano, Executive Judge of the then Court of First Instance of Rizal [Quezon City], under which the premises known as No. 19, Road 3, Project 6, Quezon City, and 784 Units C & D, RMS Building, Quezon Avenue, Quezon City, business addresses of the "Metropolitan Mail" and "We Forum" newspapers, respectively, were searched, and office and printing machines, equipment, paraphernalia, motor vehicles and other articles used in the printing, publication and distribution of the said newspapers, as well as numerous papers, documents, books and other written literature alleged to be in the possession and control of petitioner 7 4S 2014-2015  

San Beda College of Law CIVIL LAW REVIEW   Jose Burgos, Jr. publisher-editor of the "We Forum" newspaper, were seized. Petitioner avers, among others, that the seizure of the properties mentioned above amounts to seizure of real properties, which cannot be validly conducted under the strength of a search warrant. It must be noted that real properties are not susceptible of confiscation under a search warrant. Hence this appeal which assails the validity of the search and the seizure of the properties of the petitioner. ISSUE: Are the machineries seized under the disputed warrants real properties? RULING: NO. Under Article 415[5] of the Civil Code of the Philippines, "machinery, receptables, instruments or implements intended by the owner of the tenement for an industry or works which may be carried on in a building or on a piece of land and which tend directly to meet the needs of the said industry or works" are considered immovable property. In Davao Sawmill Co. v. Castillo where this legal provision was invoked, the Supreme Court ruled that machinery which is movable by nature becomes immobilized when placed by the owner of the tenement, property or plant, but not so when placed by a tenant, usufructuary, or any other person having only a temporary right, unless such person acted as the agent of the owner. In the case at bar, petitioners do not claim to be the owners of the land and/or building on which the machineries were placed. This being the case, the machineries in question, while in fact bolted to the ground remain movable property susceptible to seizure under a search warrant. LOPEZ v. OROSA, JR. and PLAZA THEATER INC. G.R. Nos. L-10817-18, 28 February 1958 [MENESES] FACTS: Petitioner Enrique Lopez agreed to supply the lumber necessary for the construction of Vicente Orosa’s theater, the latter assuming personal liability for any account that the said construction might incur. Pursuant to the verbal agreement, Lopez delivered the lumber which was used for the construction of the Plaza Theater on the land which Plaza Theater Inc. owned. The total cost of the materials was not paid. After several demands for payment of the balance and as the obligation still remained unsettled, Lopez filed a complaint with the CFI of Batangas against Orosa and Plaza Theater, Inc. In his complaint, Lopez prayed that in case of failure to pay the remaining balance for the materials, the building and the land on which the theater was constructed should be sold at a public auction and that the proceeds thereof be applied to said indebtedness. The lower court ruled in favor of Lopez, stating that Lopez acquired a materialman’s lien over the building but such lien did not extend to the land on which the construction was made. Lopez tried to secure a modification of the decision in so far as the lower court declared that the lien did not extend to the land. He contended that that the lien created in favor of the furnisher of the materials used for the construction, 8 4S 2014-2015  

San Beda College of Law CIVIL LAW REVIEW   repair or refection of a building is also extended to the land. He cited Art. 1923 of the Spanish Civil Code which reads as follows: Art. 1923. With respect to determinate real property and real rights of the debtor, the following are preferred: 5. Credits for refection, not entered or recorded, with respect to the real estate upon which the refection was made, and only with respect to other credits…

Lopez argued that in view of the employment of the phrase real estate, and inasmuch as said provision does not contain any specification delimiting the lien to the building, said article must be construed as to embrace both the land and the building or structure adhering thereto. ISSUE: Is a building considered an immovable property and therefore included in the term “real estate”, as contended by Lopez? RULING: YES. The inclusion of the term building, separate and distinct from the land, in the enumeration of what may constitute real properties under Art. 415 of the Civil Code, could mean only one thing – that a building is by itself an immovable property. In the absence of any specific provision of law to the contrary, a building is an immovable property, irrespective of whether or not said structure and the land on which it is adhered to belong to the same owner. YAP v. TANADA G.R. No. L-32917, 18 July 1988 [ORSUA] FACTS: Respondent Goulds Pumps International Inc. filed a complaint against Yap and his wife, seeking recovery of Pl,459.30 representing the balance of the price and installation cost of a water pump in the latter's premises. The City Court decided in respondent’s favor. Hence, Yap appealed to the CFI. CFI Judge Tanada also ruled in favor of respondent. Judge Tanada issued an order granting Goulds' Motion for Issuance of Writ of Execution. Judge Tanada denied Yap's motion for reconsideration and authorized execution of the judgment. In the meantime the Sheriff levied on the water pump in question, and by notice scheduled the execution sale thereof. But in view of the pendency of Yap's motion for reconsideration, suspension of the sale was directed by the judge. It appears however that a copy of this order was not transmitted to the Sheriff "through oversight, inadvertence and pressure of work" of the Branch Clerk of Court. So the Deputy Provincial Sheriff went ahead with the scheduled auction sale and sold the property levied on to Goulds as the highest bidder. Yap filed a motion to set aside execution sale and to quash alias writ of execution. He argued that the sale was made without the notice required by Sec. 18, Rule 39, of the New Rules of Court, i.e., notice by publication in case 9 4S 2014-2015  

San Beda College of Law CIVIL LAW REVIEW   of execution sale of real property, the pump and its accessories being immovable because attached to the ground with character of permanency (Art. 415, Civil Code). Yap's motion was thereafter denied. Judge Tanada pointed out that the motion had "become moot and academic" since the decision received by the defendant had long become final when the order for the issuance of a writ of execution was promulgated. ISSUE: Is the water pump involved an immovable property as what petitioner claimed as a basis for annulling the execution sale? RULING: NO. The Civil Code considers as immovable property, among others, anything “attached to an immovable in a fixed manner, in such a way that it cannot be separated therefrom without breaking the material or deterioration of the object.” The pump does not fit this description. It could be, and was in fact separated from Yap’s premises without being broken or suffering deterioration. Obviously the separation or removal of the pump involved nothing more complicated than the loosening of bolts or dismantling of other fasteners. MACHINERY AND ENGINEERING SUPPLIES, INC v. COURT OF APPEALS, G.R. No. L-7057, 29 October 1954 [PAGAYANAN] FACTS: Petitioner filed a complaint for replevin in CFI of Manila for the recovery of the machineries and equipment delivered to Ipo Limestone Co, Inc. at their factory in barrio Bigti, Norzagaray, Bulacan. Upon application ex-parte of the petitioner, accompanied by the affidavit of Ramon S. Roco, president of petitioner company, and upon approval of petitioner's bond, respondent judge issued an order, commanding the Provincial Sheriff of Bulacan to seize and take immediate possession of the properties specified in the order. Two deputy sheriffs of Bulacan, and other technical men and laborers proceeded to Bigti, for the purpose of carrying the court's order into effect. Leonardo Contreras, manager of the respondent Company, and Pedro Torres, in charge thereof, met the deputy sheriffs, and Contreras handed to them a letter addressed to Atty. Palad, protesting against the seizure of the properties in question, on the ground that they are not personal properties. Contending that the Sheriff's duty is merely ministerial, the deputy sheriffs, Roco, the latter's crew of technicians and laborers, Contreras and Torres, went to the factory. Roco's attention was called to the fact that the equipment could not possibly be dismantled without causing damages or injuries to the wooden frames attached to them. As Roco insisted in dismantling the equipment on his own responsibility, alleging that the bond was posted for such eventuality, the deputy sheriffs directed that some of the supports thereof be cut.

10 4S 2014-2015  

San Beda College of Law CIVIL LAW REVIEW   ISSUE: Can the machineries, which could not be dismantled without causing damages or injuries to the frames attached, be the proper subjects of a writ of replevin? RULING: NO. When the sheriff repaired to the premises of respondent, Ipo Limestone Co., Inc., the machinery and equipment in question appeared to be attached to the land, particularly to the concrete foundation of said premises, in a fixed manner, in such a way that the former could not be separated from the latter "without breaking the material or deterioration of the object." Hence, in order to remove said outfit, it became necessary, not only to unbolt the same, but, also, to cut some of its wooden supports. Moreover, said machinery and equipment were "intended by the owner of the tenement for an industry" carried on said immovable and tended "directly to meet the needs of the said industry." For these reasons, they were already immovable property pursuant to paragraphs 3 and 5 of Article 415 of Civil Code of the Philippines, which are substantially identical to paragraphs 3 and 5 of Article 334 of the Civil Code of Spain. As such immovable property, they were not subject to replevin.

CLASSIFICATION OF PROPERTY: Property in Relation to Whom it Belongs   SALVADOR H. LAUREL v. RAMON GARCIA G.R. No. 92013, 25 July 1990 [PARUNGAO] FACTS: The properties and the capital goods and services procured by the Philippines under the Reparation Agreement for National Development projects are part of the indemnification to the Filipino people for their losses in life and property and their suffering during World War II. The Roppongi property consists of the land and building specifically designated under the Agreement “for the chancery of the Philippine Embassy.’’ Due to the failure of the Philippine Government to provide necessary funds, the property has remained undeveloped. The Executive Branch has been pushing, with great vigor, its decision to sell the reparation properties starting with the Roppongi lot. ISSUEs: 1. Can the Roppongi property and others of its kind be alienated by the Philippine Government? 2. Has the Roppongi property become patrimonial? RULING: 1. As property of public dominion, the Roppongi lot is outside the commerce of man. It cannot be alienated. Its ownership is a special collective ownership 11 4S 2014-2015  

San Beda College of Law CIVIL LAW REVIEW   for general use and enjoyment, an application to the satisfaction of collective needs, and resides in the social group. The purpose is not to serve the State as a juridical person, but the citizens; it is intended for the common and public welfare and cannot be the object of appropriation. The Roppongi property is correctly classified under paragraph 2 of Article 420 as property belonging to the State and intended for some public service. 2. The fact that the Roppongi site has not been used for a long time for actual Embassy service does not automatically convert it to patrimonial property. Any such conversion happens only if the property is withdrawn from public use. A property continues to be part of the public domain, not available for private appropriation or ownership until there is a formal declaration on the part of the government to withdraw it from being such. RABUCO v. VILLEGAS G.R. No. L-24661, 28 February1974 [PULMA] FACTS: The Court herein upholds the constitutionality of Republic Act 3120 on the strength of the established doctrine that the subdivision of communal land of the State (although titled in the name of the municipal corporation) and conveyance of the resulting subdivision lots by sale on installment basis to bona fide occupants by Congressional authorization and disposition does not constitute infringements of the due process clause or the eminent domain provisions of the Constitution but operates simply as a manifestation of the legislature's right of control and power to deal with State property. The petitioners are the tenants of the land in dispute who are being ejected on account of Republic Act 3120. In Republic Act 3120 Congress converted the lot in question together with another lot in San Andres, Malate "which are reserved as communal property" into "disposable or alienable lands of the State to be placed under the administration and disposal of the Land Tenure Administration" for subdivision into small lots not exceeding 120 square meters per lot for sale on installment basis to the tenants or bona fide occupants thereof. The respondent officials also contend that the act is unconstitutional for depriving the City of Manila of the lots in question without just compensation. ISSUE: Is R.A. 3120 Constitutional?   RULING: YES. Respondents city officials' contention that the Act must be stricken down as unconstitutional for depriving the city of Manila of the lots in question and providing for their sale in subdivided small lots to bona fide occupants or tenants without payment of just compensation is untenable and without basis, since the lots in question are manifestly owned by the city in its public and governmental capacity and are therefore public property over which Congress had absolute control as distinguished from patrimonial property owned by it in 12 4S 2014-2015  

San Beda College of Law CIVIL LAW REVIEW   its private or proprietary capacity of which it could not be deprived without due process and without just compensation. Here, Republic Act 3120 expressly declared that the properties were "reserved as communal property" and ordered their conversion into "disposable and alienable lands of the State" for sale in small lots to the bona fide occupants thereof. It is established doctrine that the act of classifying State property calls for the exercise of wide discretionary legislative power which will not be interfered with by the courts. The Court therein reaffirmed the established general rule that "regardless of the source or classification of land in the possession of a municipality, excepting those acquired with its own funds in its private or corporate capacity, such property is held in trust for the State for the benefit of its inhabitants, whether it be for governmental or proprietary purposes. It holds such lands subject to the paramount power of the legislature to dispose of the same, for after all it owes its creation to it as an agent for the performance of a part of its public work, the municipality being but a subdivision or instrumentality thereof for purposes of local administration. Accordingly, the legal situation is the same as if the State itself holds the property and puts it to a different use" and stressed that "the property, as has been previously shown, was not acquired by the City of Manila with its own funds in its private or proprietary capacity. That it has in its name a registered title is not questioned, but this title should be deemed to be held in trust for the State as the land covered thereby was part of the territory of the City of Manila granted by the sovereign upon its creation." MACASIANO v. DIOKNO G.R. No. 97764, 10 August 1992 [REMIGIO] FACTS: On June 13, 1990, the Municipality of Paranaque passed Ordinance No. 86, Series of 1990 which authorized the closure of J. Gabriel, G.G. Cruz, Bayanihan, Lt. Garcia Extension and Opena Streets located at Baclaran, Parañaque, Metro Manila and the establishment of a flea market thereon. The said ordinance was approved by the municipal council pursuant to MMC Ordinance No. 2, Series of 1979, authorizing and regulating the use of certain city and/or municipal streets, roads and open spaces within Metropolitan Manila as sites for flea market and/or vending areas, under certain terms and conditions. On June 20, 1990, the municipal council of Parañaque issued a resolution authorizing Parañaque Mayor Walfrido N. Ferrer to enter into contract with any service cooperative for the establishment, operation, maintenance and management of flea markets and/or vending areas. On August 8, 1990, respondent municipality and respondent Palanyag, a service cooperative, entered into an agreement whereby the latter shall operate, maintain and manage the flea market in the aforementioned streets with the obligation to 13 4S 2014-2015  

San Beda College of Law CIVIL LAW REVIEW   remit dues to the treasury of the municipal government of Parañaque. Consequently, market stalls were put up by respondent Palanyag on the said streets. ISSUE: Is an ordinance or resolution Issued by the municipal council of Parañaque authorizing the lease and use of public streets or thoroughfares as sites for flea markets valid? RULING: Article 424 of the Civil Code lays down the basic principle that properties of public dominion devoted to public use and made available to the public in general are outside the commerce of man and cannot be disposed of or leased by the local government unit to private persons. Properties of the local government which are devoted to public service are deemed public and are under the absolute control of Congress. Hence, local governments have no authority whatsoever to control or regulate the use of public properties unless specific authority is vested upon them by Congress. When it is already withdrawn from public use, the property then becomes patrimonial property of the local government unit concerned. It is only then that the respondent municipality can "use or convey them for any purpose for which other real property belonging to the local unit concerned might be lawfully used or conveyed" in accordance with the last sentence of Section 10, Chapter II of Blg. 337, known as Local Government Code. REPUBLIC v. COURT OF APPEALS G.R. No. 100709, 14 November 1997 [RAMPAS] FACTS: Josefina Morato, private respondent, applied sometime in 1972 a free patent on a parcel of land, it was approved and it specifically stated that the same cannot be sold or encumbered for 5 years. It was however leased and later on, it was discovered that part of the land is a foreshore land. Petitioner moved for the cancellation of the free patent. ISSUE: Is the questioned land a foreshore land and thus must be reverted to the public domain? RULING: YES. When the sea moved towards the estate and the tide invaded it, the invaded property became foreshore land and passed the realm of the public domain and accordingly cannot be a subject of a free patent. (Article 420)

14 4S 2014-2015  

San Beda College of Law CIVIL LAW REVIEW   PROVINCE OF ZAMBOANGA DEL NORTE v. CITY OF ZAMBOANGA, et al. L-24440, 28 March 1968 [SABILALA] FACTS: After Zamboanga Province was divided into two (Zamboanga del Norte and Zamboanga del Sur), Republic Act 3039 was passed providing that — “All buildings, properties, and assets belonging to the former province of Zamboanga and located within the City of Zamboanga are hereby transferred free of charge in favor of the City of Zamboanga.'' Suit was brought alleging that this grant without just compensation was unconstitutional because it deprived the province of property without due process. Included in the properties were the capital site and capitol building, certain school sites, hospital and leprosarium sites, and high school playgrounds. ISSUES: 1. Are the properties mentioned, properties for public use or patrimonial? 2. Should the city pay for said properties? RULING: 1. If we follow the Civil Code classification, only the high school playgrounds are for public use (in the sense that generally, they are available to the general public), and all the rest are PATRIMONIAL (since they are not devoted to public use but to public service; since they are not for public use, under Art. 424 of the Civil Code, they are patrimonial. [NOTE: For public use if ANYBODY can use; for public service if only AUTHORIZED persons can use.]. [NOTE: Had they been owned by the STATE, they would not have been patrimonial but would have been properties of public dominion — for this would include public service, conformably with Art. 420, par. 2.]. BUT if we follow the law of Municipal Corporations (and not the Civil Code), as long as the purpose is for a public service (governmental service like public education, public health, local administration), the property should be considered for PUBLIC USE. 2. If the Civil Code classification is used, since almost all the properties involved are patrimonial, the law would be unconstitutional since the province would be deprived of its own property without just compensation. If the law on Municipal Corporations would be followed, the properties would be of public dominion, and therefore NO COMPENSATION would be required.

15 4S 2014-2015  

San Beda College of Law CIVIL LAW REVIEW   It is this law on Municipal Corporations that should be followed. Firstly, while the Civil Code may classify them as patrimonial, they should not be regarded as ordinary private property. They should fall under the control of the State, otherwise certain governmental activities would be impaired. Secondly, Art. 424, 2nd paragraph itself says “without prejudice to the provisions (or PRINCIPLES) of special laws.” CHAVEZ v. PUBLIC ESTATES AUTHORITY G.R. No. 133250, July 9, 2002 [SANTILLAN] FACTS: President Marcos issued Presidential Decree No. 1084 creating PEA. PD No. 1084 tasked PEA “to reclaim land, including foreshore and submerged areas,” and “to develop, improve, acquire, x x x lease and sell any and all kinds of lands.” Then President Aquino issued Special Patent, granting and transferring to PEA “the parcels of land so reclaimed under the Manila-Cavite Coastal Road and Reclamation Project (MCCRRP).” Subsequently, the Register of Deeds issued Transfer Certificates of Title in the name of PEA, covering the three reclaimed islands known as the “Freedom Islands”. PEA entered into a Joint Venture Agreement with AMARI, a private corporation, to develop the Freedom Islands. Under the amended JVA it required the reclamation of an additional 250 hectares of submerged areas surrounding these islands and PEA is obligated to transfer to AMARI the latter’s seventy percent proportionate share in the reclaimed areas as the reclamation progresses. PEA and AMARI entered into the JVA through negotiation without public bidding. The JVA was subsequently approved, hence, this appeal to assail the constitutionality of the JVA. ISSUE: Can a private corporation acquire and own reclaimed foreshore and submerged areas in Manila Bay? RULING: The JVA contravenes Section 3, Article XII of the 1987 Constitution which provides that private corporations “shall not hold such alienable lands of the public domain except by lease.” The transfer of title and ownership to AMARI clearly means that AMARI will “hold” the reclaimed lands other than by lease. Furthermore, since the Amended JVA also seeks to transfer to AMARI ownership of 290.156 hectares of still submerged areas of Manila Bay, such transfer is void for being contrary to Section 2, Article XII of the 1987 Constitution which prohibits the alienation of natural resources other than agricultural lands of the public domain The Regalian doctrine is deeply implanted in our legal system. Foreshore and submerged areas form part of the public domain and are inalienable. Lands reclaimed from foreshore and submerged areas also form part of the public domain and are also inalienable, unless converted pursuant to law into alienable or disposable lands of the public domain. Historically, lands reclaimed by the government are sui generis, not available for sale to private parties unlike other alienable public lands. Reclaimed lands retain their 16 4S 2014-2015  

San Beda College of Law CIVIL LAW REVIEW   inherent potential as areas for public use or public service. Alienable lands of the public domain, increasingly becoming scarce natural resources, are to be distributed equitably among our ever-growing population. To insure such equitable distribution, the 1973 and 1987 Constitutions have barred private corporations from acquiring any kind of alienable land of the public domain. NOTES: DENR is vested with the power to authorize the reclamation of areas under water, while PEA is vested with the power to undertake the physical reclamation of areas under water, whether directly or through private contractors. DENR is also empowered to classify lands of the public domain into alienable or disposable lands subject to the approval of the President. On the other hand, PEA is tasked to develop, sell or lease the reclaimed alienable lands of the public domain.

·

·

·

·

Applicable CIVIL CODE Provisions: ART 420 and 422 Summary: 1. The 157.84 hectares of reclaimed lands comprising the Freedom Islands, now covered by certificates of title in the name of PEA, are alienable lands of the public domain. PEA may lease these lands to private corporations but may not sell or transfer ownership of these lands to private corporations. PEA may only sell these lands to Philippine citizens, subject to the ownership limitations in the 1987 Constitution and existing laws. 2. The 592.15 hectares of submerged areas of Manila Bay remain inalienable natural resources of the public domain until classified as alienable or disposable lands open to disposition and declared no longer needed for public service. The government can make such classification and declaration only after PEA has reclaimed these submerged areas. Only then can these lands qualify as agricultural lands of the public domain, which are the only natural resources the government can alienate. In their present state, the 592.15 hectares of submerged areas are inalienable and outside the commerce of man. 3. Since the Amended JVA seeks to transfer to AMARI, a private corporation, ownership of 77.34 hectares of the Freedom Islands, such transfer is void for being contrary to Section 3, Article XII of the 1987 Constitution which prohibits private corporations from acquiring any kind of alienable land of the public domain. 4. Since the Amended JVA also seeks to transfer to AMARI ownership of 290.156 hectares of still submerged areas of Manila Bay, such transfer is void for being contrary to Section 2, Article XII of the 1987 Constitution which prohibits the alienation of natural resources other than agricultural lands of the public domain. PEA may reclaim these submerged areas. Thereafter, the government can classify the reclaimed lands as alienable or disposable, and further declare them no longer needed for public service. Still, the transfer of such reclaimed alienable lands of the public domain to AMARI will be void in view of Section 3, Article XII of the 1987 Constitution which prohibits private corporations from acquiring any kind of alienable land of the public domain.

VILLARICO v. SARMIENTO G.R. No. 136438, 11 November 2004 [SAYO] FACTS: Petitioner is the owner of a lot covered by TCT No. 95453. Petitioner’s lot is separated from the Ninoy Aquino Avenue highway by a strip of land belonging to the government. As the highway is elevated, the DPWH constructed stairways at several portions of this strip of public land to enable the people to have access to the highway. Respondents had a building constructed on a portion of the said government land. Subsequently, by means of Deed of Exchange of Real Property, petitioner acquired the same area of the government land and was registered in petitioner’s name as TCT. No. 74430. Petitioner filed a complaint for accion publiciana against respondents alleging that respondents’ structures on the government land closed his “right of way” to the Ninoy Aquino Avenue; and encroached on a portion of his lot covered by TCT No. 74430. The RTC ruled that respondents have a better right of 17 4S 2014-2015  

San Beda College of Law CIVIL LAW REVIEW   possession on the portion of the government land except for the portion covered by TCT No. 74430. The CA affirmed in toto. Hence, this petition. ISSUES: 1. Does petitioner have a better right over the portion of the government land that closed petitioner’s right of way? 2. Is the petitioner entitled to possession over the portion that encroached the land covered by TCT No. 74430? RULING: 1. NO. It is not disputed that the lot on which petitioner’s alleged “right of way” exists belongs to the state or property of public dominion. Public use is “use that is not confined to privileged individuals but is open to the indefinite public.” Considering that the lot on which the stairways were constructed is a property of public dominion, it cannot be burdened by a voluntary easement of right of way in favor of herein petitioner. In fact, its use by the public is mere tolerance of the government through the DPWH. Petitioner cannot appropriate it for himself. Verily, he cannot claim any right of possession over it. Neither do the respondents have a better right of possession over the same. 2. YES. Being its owner, he is entitled to its possession.

OWNERSHIP: Rights Included in Ownership   JAVIER v. VERIDIANO II G.R. No. L-48050, 10 October 1994 [SENTILLAS] FACTS: Sometime in January 1963 petitioner filed a Miscellaneous Sales Application for Lot No. 1641; Sometime in December 1970, alleging that she was forcibly dispossessed of a portion of the land by a certain Ben Babol, petitioner instituted a complaint for forcible entry before the City Court of Olongapo City; On 7 November 1972 the City Court of Olongapo City dismissed the case on the ground that "it appears to the Court that the Bureau of Lands has considered the area in question to be outside Lot 1641 of the plaintiff. . . ." and became final and executory on 30 April 1973; Subsequently, on 17 December 1973, petitioner was granted Miscellaneous Sales Patent No. 5548 and issued Original Certificate of Title No. P-3259 covering Lot No. 1641. Meanwhile, Ben Babol sold the property he was occupying, including the portion of about 200 square meters in question, to a certain Reino Rosete.

18 4S 2014-2015  

San Beda College of Law CIVIL LAW REVIEW   On 29 June 1977, or after about four (4) years from the finality of the dismissal of Civil Case No. 926, petitioner instituted a complaint for quieting of title and recovery of possession with damages against Ben Babol and Reino Rosete. Instead of filing a responsive pleading, therein defendant Reino Rosete (private respondent herein) moved to dismiss the complaint on the ground of res judicata. ISSUE: Is there identity of causes of action if the first case is an ejectment suit and the second, an accion reivindicatoria? Ruling: NO. The first case is a complaint for forcible entry, where what is at Issue is prior possession, regardless of who has lawful title over the disputed property. The only Issue in an action for forcible entry is the physical or material possession of real property, that is, possession de facto and not possession de jure. A judgment rendered in a case for recovery of possession is conclusive only on the question of possession and not on the ownership. It does not in any way bind the title or affect the ownership of the land or building. On the other hand, the second case is in reality an action to recover a parcel of land or an accion reivindicatoria under Art. 434 of the Civil Code, and should be distinguished from the first case, which is an accion interdictal. Accion reivindicatoria or accion de reivindicacion is an action whereby plaintiff alleges ownership over a parcel of land and seeks recovery of its full possession. It is different from accion interdictal or accion publiciana where plaintiff merely alleges proof of a better right to possess without claim of title. BUSTOS v. COURT OF APPEALS G.R. No. 120784-85, January 24, 2001 [SINOCRUZ] FACTS: Paulino Fajardo died intestate leaving 4 children: Manuela, Trinidad, Beatriz and Marcial. The heirs executed an extra-judicial partition of the estate of Paulino. On the same day, Manuela sold her share (Lot 248) to Moses Mendoza, Beatriz’s husband, by a deed of sale. Trinidad, however, was in physical possession of the land and refused to surrender the same to Moses. Moses then filed a complaint for partition claiming the ¼ share of Manuela which was sold to him. During the pendency of hearing, Trinidad died and so her heirs executed an extra-judicial partition of her estate which included Lot 248. After which, one of Trinidad’s heir sold Lot 248 to spouses Viray.

19 4S 2014-2015  

San Beda College of Law CIVIL LAW REVIEW   The trial court, nonetheless, rendered a decision in favor of Moses, granting the partition and segregating the ¼ portion. By virtue of such decision, Moses subsequently sold the Lot 248 to spouses Warlito and Herminia Bustos who were lessees of Trinidad’s husband. Since the spouses Bustos were in actual possession of the subject land, the Virays, claiming to be rightful buyers Lot 248, filed with the MCTC an action for unlawful detainer against them. The MCTC decided in favor of the Virays and issued writs of execution and demolition against the spouses Bustos. The spouses Bustos, however, filed a petition for certiorari, prohibition and injunction which stayed the aforementioned writs. ISSUE: Can petitioner-spouses Bustos be ejected from what is now their own land? Ruling: NO. An owner who cannot exercise the seven (7) “juses” or attributes of ownership- the right to possess, to use and enjoy, to abuse or consume, to accessories, to dispose or alienate, to recover or vindicate and to the fruits is a crippled owner. In the unlawful detainer case, the Court of Appeals affirmed the decision of the trial court as to possession on the ground that the decision has become final and executory. Thus, petitioners may be evicted. However, in the accion reinvidicatoria, the Court of Appeals affirmed the ownership of spouses Bustos over the subject land. Hence, the court declared spouses Bustos as the lawful owners of the land. The stay of the execution in the unlawful detainer case is warranted by the fact that spouses Bustos are now legal owners of the land. To eject them from the land they owned would certainly result in injustice. Besides, the issue of possession was rendered moot when the ownership was adjudicated to the spouses Bustos by virtue of a deed of sale. Placing them in possession is the necessary and logical consequence of such decision declaring them the rightful owners. HEIRS OF ROMAN SORIANO v. COURT OF APPEALS G.R. No. 128177, 15 August 2001 [TILOS] FACTS: A piece of land located in Lingayen, Pangasinan is the disputed property in this case. Said land was originally owned by one Adriano Soriano, subsequently it was leased for a period of 15 years to the Spouses David and Consuelo with RAMON SORIANO, son of Adriano and herein petitioner, acting as caretaker/tenant of the property during the duration of the lease. Upon the death of Adriano the lot he owned was divided into TWO and given to his heirs. One of the lots inherited was sold to the Spouses ABALOS, here. The other lot was also bought by the Spouses Abalos although not completely 20 4S 2014-2015  

San Beda College of Law CIVIL LAW REVIEW   (only ¾ of the lot). The lots in question were subsequently registered in the name of the Spouses Abalos. The courts later declared them to be the undisputed owners thereof. Soriano questions their ownership of the land and so filed cases against the spouses. Currently Soriano is still in possession of the land claiming rights of “Security of Tenure” as a tenant of the land. ISSUE: May a winning party in a land registration case effectively eject the possessor thereof, whose security of tenure rights are still pending determination before the DARAB? RULING: NO. Possession and ownership are distinct legal concepts. Possession is the holding of a thing or the enjoyment of a right. Literally, to possess means to actually and physically occupy a thing with or without right. A judgment of ownership does not necessarily include possession as a necessary incident. Such declaration pertains only to OWNERSHIP and does not automatically include possession. This is especially true in the case at bar wherein petitioner is occupying the land allegedly in the concept of an agricultural tenant. The court says “allegedly” due to the fact that there is still a pending case in the DARAB (Department of Agrarian Reform and Adjudication Board) on the Issue. The Issue of ownership of the subject land has been laid to rest by final judgment; however the right of possession is yet to be resolved. The Tenancy Act, which protects the rights of agricultural tenants, may limit the exercise of rights by the lawful owners. The exercise of the rights of ownership yields to the exercise of the rights of an agricultural tenant. Since the rights of Soriano to possess the land are still pending litigation in the DARAB he is protected from dispossession of the land until final judgment of said court unless Soriano’s occupancy is found by the court to be unlawful. DE GARCIA v. COURT OF APPEALS G.R. No. L-20264, 30 January 1971 FACTS: Angelina D. Guevarra seeks recovery from Consuelo de Garcia of one lady’s diamond ring 18cts.white gold mounting, with one 2.05 cts. diamond-solitaire, and four brills 0.10 cts. total weight, which she bought on October 27, 1947 from R. Rebullida, Inc. Respondent Guevarra’s evidence tend to show that while talking to petitioner Garcia, she recognized her ring on the finger of Mrs. Garcia and inquired where she bought it. Guevarra explained that the ring was stolen from her house. The ring fitted Guevarra’s finger. Upon examination conducted by Mr. Rebullida with the aid of high power lens and after consulting the stock card thereon, concluded that it was the very ring that Guevarra bought from him in 1947. The ring was returned to Garcia who despite a written request failed to deliver the ring to Guevarra. Later on, when sheriff tried to serve the writ of seizure (replevin), Garcia refused to deliver the ring which had been examined by Mr. Rebullida, claiming it was lost.

21 4S 2014-2015  

San Beda College of Law CIVIL LAW REVIEW   ISSUES: 1. Can Respondent Angelina D. Guevarra recover the diamond ring which is the subject of the dispute from petitioner Consuelo D. de Garcia? 2. Is Consuelo de Garcia’s alleged possession in good faith of the diamond ring in question equivalent to title? RULING: 1. YES. The controlling provision is Article 559 of the Civil Code. It reads: “The possession of movable property acquired in good faith is equivalent to title. Nevertheless, one who has lost any movable or has been unlawfully deprived thereof may recover it from the person in possession of the same. If the possessor of a movable lost of which the owner has been unlawfully deprived, has acquired it in good faith at a public sale, the owner cannot obtain its return without reimbursing the price paid therefor.” Respondent Angelina D. Guevarra, having been unlawfully deprived of the diamond ring in question, was entitled to recover it from petitioner Consuelo S. de Garcia who was found in possession of the same. The only exception the law allows is when there is acquisition in good faith of the possessor at a public sale, in which case the owner cannot obtain its return without reimbursing the price. 2. NO. Possession in good faith does not really amount to title, for the reason that Article 1132 of the Civil Code provides for a period of acquisitive prescription for movables through “uninterrupted possession for four years in good faith. The title established by the first clause of Article 559 is only a presumptive title sufficient to serve as a basis for acquisitive prescription, that the clause immediately following provides that “one who has lost any movable or has been unlawfully deprived thereof, may recover it from the person in possession of the same.” RODIL ENTERPRISES, INC. v. COURT OF APPEALS, CARMEN BONDOC, TERESITA BONDOC-ESTO, DIVISORIA FOOTWEAR and CHUA HUAY SOON G.R. No. 129609, 29 November 2001 [ABIERA] FACTS: Petitioner Rodil Enterprises (Rodil) is the lessee of the Ides O’Racca building (O’Racca) since 1959. It was a former alien property over which the Republic acquired ownership by virtue of RA 477. Rodil entered into a sublease contract with respondents Bondoc, Bondoc-Esto, Divisoria Footware and Chua Huay Soon, members of O’Racca Building Tenants Association, Inc. (Association). On January 8, 1987, Rodil offered to purchase the property conformably with BP 233. On July 22, 1987, the Association also offered to lease the same building through Department of General Services and Real Estate Property Management (DGSREPM). Pending action on the offer of Rodil to purchase the property, the Republic granted Rodil’s request for the renewal of the lease contract on Sept. 23, 1987 for 5 more years. The renewal contract however was disapproved by the DGSREPM Secretary De Jesus 22 4S 2014-2015  

San Beda College of Law CIVIL LAW REVIEW   upon recommendation of DGSREPM Undersecretary Banas on the ground that the offer of the Association was more beneficial to the Republic. On October 1987, Rodil filed an action to enjoin the Association from collecting rentals from the occupants or sub-lessees of O’Racca. This was granted by the trial court and upheld by the CA. On May 18, 1992, Rodil signed a renewal contract for 10 more years of lease, which was approved by the DENR Secretary. The Association then filed an action to set aside the renewal contract, but the same was denied by the trial court. Rodil then filed an action for unlawful detainer against herein respondents. The MeTC upheld Rodil’s right to eject, which was then upheld by the RTC. While the consolidated appeals were pending, the CA 2nd Division declared the renewal contract between Rodil and the Republic as null and void. Rodil moved for reconsideration but the same was denied which prompted it to file an action for certiorari. The CA 4th Division likewise set aside the MeTC and the RTC’s decision and dismissed the action of Rodil for unlawful detainer. ISSUES: 1. Did the Court of Appeals err in annulling petitioner’s renewal contract with the Republic? 2. May petitioner validly eject herein respondents even though the former is not in actual possession of the property? RULING: 1. YES. The owner has the right to enjoy and dispose of a thing, without other limitations than those established by law. Every owner has the freedom of disposition over his property. It is an attribute of ownership, and this rule has no exception. The Republic being the owner of the disputed property enjoys the prerogative to enter into a lease contract with Rodil in the exercise of its jus disponendi. 2. YES. In an action for unlawful detainer, the plaintiff need not have been in prior physical possession. Respondents have admitted that they have not entered into any lease contract with the Republic and that their continued occupation of the subject property was merely by virtue of acquiescence. Since the occupation of respondents was merely tolerated by the Republic, the right of possession of the latter remained uninterrupted. It could therefore alienate the same to anyone it choose. Unfortunately for respondents, the Republic chose to alienate the subject premises to Rodil by virtue of a contract of lease entered into on May 18, 1992. Resultantly, the petitioner had the right to file the action for unlawful detainer against respondents as one from whom possession of property has been unlawfully withheld.

23 4S 2014-2015  

San Beda College of Law CIVIL LAW REVIEW   CORNELIO M. ISAGUIRRE v. FELICITAS DE LARA G.R. No. 138053, 31 May 2000 [ALDUESO] FACTS: Alejandro de Lara was the original applicant-claimant for a Miscellaneous Sales Application over a parcel of land filed with the Bureau of Lands. Upon his death, he, was succeeded by his wife - respondent Felicitas de Lara, as claimant. By virtue of a decision rendered by the Secretary of Agriculture and Natural Resources, a subdivision survey was made and the area was reduced to 1,000 square meters. On this lot stands a two-story residential-commercial apartment declared for taxation purposes under TD 43927 in the name of respondent’s sons - Apolonio and Rodolfo, both surnamed de Lara. Felicitas obtained a loan from PNB. When she encountered financial difficulties, respondent approached petitioner Cornelio M. Isaguirre. A document denominated as "Deed of Sale and Special Cession of Rights and Interests" was executed by respondent and petitioner, whereby the former sold a 250 square meter portion of Lot No. 502, together with the two-story commercial and residential structure standing thereon, in favor of petitioner, for and in consideration of the sum of P5,000. Subsequently, Apolonio and Rodolfo de Lara filed a complaint against petitioner for recovery of ownership and possession of the two-story building but was dismissed. Isaguirre filed a sales application over the subject property on the basis of the deed of sale. His application was approved resulting in the issuance of OCT No. P-11566. Meanwhile, the sales application of respondent over the entire 1,000 square meters of subject property (including the 250 square meter portion claimed by petitioner) was also given due course, resulting in the issuance of OCT No. P-13038, in the name of respondent. Due to the overlapping of titles, petitioner filed an action for quieting of title and damages with the RTC of Davao City against respondent. The RTC rendered judgment in favor of petitioner declaring him to be the lawful owner of the disputed property. However, the Court of Appeals reversed the trial court’s decision, holding that the transaction entered into by the parties was an equitable mortgage, not a sale. As a consequence of its decision, the appellate court also declared OCT No.P-11566 issued in favor of petitioner to be null and void. In a case docketed as G. R. No. 120832, this Court affirmed the decision of the Court of Appeals and the petitioner’s MR was subsequently denied. Petitioner contends that the CA has acted with grave abuse of discretion when it did not held that the RTC erred in issuing a writ of possession as neither the CA decision nor the SC decision mentioned, ordered immediate delivery of possession to the respondent.

24 4S 2014-2015  

San Beda College of Law CIVIL LAW REVIEW   ISSUES: Is the trial court correct in granting writ of possession against petitioner when the court found that the respondent is the owner of said property? RULING: YES. As the sole owner, respondent has the right to enjoy her property, without any other limitations than those established by law.Corollary to such right, respondent also has the right to exclude from the possession of her property any other person to whom she has not transmitted such property.The issuance of the writ of possession is but a necessary consequence of this Court ruling in G.R. No 120832 affirming the validity of the original certificate of title (OCT No. P-13038) in the name of respondent Felicitas de Lara, while at the same time nullifying the original certificate of title (OCT No. P-11566) in the name of petitioner Cornelio Isaguirre. Possession is an essential attribute of ownership; thus, it would be redundant for respondent to go back to court simply to establish her right to possess subject property. Contrary to petitioner’s claims, the issuance of the writ of possession by the trial court did not constitute an unwarranted modification of our decision in G.R. No. 120832, but rather, was a necessary complement thereto. It bears stressing that a judgment is not confined to what appears upon the face of the decision, but also those necessarily included therein or necessary thereto. CUSTODIO v. COURT OF APPEALS G.R. No. 116100, 9 February 1996 [ALON] FACTS: Pacifico Mabasa owns a parcel of land with a two-door apartment erected thereon situated at Interior P. Burgos St., Palingon, Tipas, Taguig, Metro Manila. Said property may be described to be surrounded by other immovable (row of houses and a septic tank) pertaining to defendants Sps. Custodio and Sps. Santos. There were two passageways available to be able to arrive at Mabasa’s property that shall pass in between the previously mentioned row of houses and septic tank. Sometime in February, 1982, one of the tenants of Mabasa vacated the apartment and when plaintiff Mabasa went to see the premises, he saw that there had been built an adobe fence in the first passageway making it narrower in width; another passageway was entirely enclosed upon the construction of an extended adobe fence. It was then that the remaining tenants of said apartment vacated the area. Pacifico Mabasa, thereafter, filed a civil case for the grant of an easement of right of way against the defendants. The decision granted Pacifico Mabasa the right of way with corresponding damages. ISSUES: Was the award of damages properly awarded? RULING: NO. The mere fact that the plaintiff suffered losses does not give rise to a right to recover damages. Whatever injury or damage may have been sustained by Mabasa by reason of the rightful use of the said land by petitioners is 25 4S 2014-2015  

San Beda College of Law CIVIL LAW REVIEW   damnum absque injuria. A person has a right to the natural use and enjoyment of his own property, according to his pleasure, for all the purposes to which such property is usually applied. As a general rule, therefore, there is no cause of action for acts done by one person upon his own property in a lawful and proper manner, although such acts incidentally cause damage or an unavoidable loss to another, as such damage or loss is damnum absque injuria. The act of petitioners in constructing a fence within their lot is a valid exercise of their right as owners, hence not contrary to morals, good customs or public policy. The law recognizes in the owner the right to enjoy and dispose of a thing, without other limitations than those established by law. PACENCIO ABEJARON, REPRESENTED BY HIS ATTORNEY-IN-FACT, ALEJANDRO ABEJARON v. FELIX NABASA AND THE COURT OF APPEALS G.R. No. 84831, 20 June 2001 [BELARMINO] FACTS: Petitioner Pacencio Abejaron alleges that he is the actual and lawful possessor and claimant of a 118 square meter portion of a 175 square meter residential lot in Silway, General Santos City. Abejaron and his family started occupying the 118 square meter land in 1945. They constructed a fence around the area and build thereon their family home. At that time, the land had not yet been surveyed. In 1949, petitioner improved their abode to become a two-story house made of wood and nipa roofing. He also intoduced improvements including a store, 5 coconut trees and avocado and banana trees. Despite the improvement introduced on the said property, Felix Nabasa, respondent, did not oppose or complain about the improvements. Knowing that the land was public in character, he declared only his house for taxation purposes. Beginning 1955, Nabasa resided on the remaining 57 square meter portion of the lot and built his house about 4 meters away from Abejaron’s house. A survery by the Bureau of Land was thereafter conducted on the area in controversy. Abejaron merely watched the survey and did not bother to apply for title to the land on the belief that he could not secure title over it as it was government property. Nabasa, however, without the knowledge and consent of Abejaron, “clandestinely, willfully, fraudulently, and unlawfully applied for and caused the titling in his name” of the entire 175 square meter lot. Petitioner imputes bad faith on the part of Nabasa because he represented himself to be the actual and lawful possessor of the entire lot despite knowledge of Abejaron’s actual occupation and possession. On September 24, 1974, an Original Certificate of Title was issued to Nabasa. An action for reconveyance was thereafter commenced by Abejaron against Nabasa. ISSUE: Is it necessary, for an action for reconveyance to prosper, that the petitioner be the absolute owner of the land in controversy?

26 4S 2014-2015  

San Beda College of Law CIVIL LAW REVIEW   RULING: YES. It is well-settled that reconveyance is a remedy granted only to the owner of the property alleged to be erroneously titled in another’s name. For an action for reconveyance based on fraud to prosper, it is essential for the party seeking reconveyance to prove by clear and convincing evidence his title to the property and the fact of fraud. Petitioner does not claim to own the land in dispute. He in fact admits that he believed the land in dispute was public in character, thus he did not declare it for taxation purposes despite possession of it for a long time. Neither did he apply for title over it on the mistaken belief that he could not apply for title of a public land. Petitioner’s evidence does not constiture the “well-nigh incontrovertible” evidence necessary to acquire title through possession and occupation of the disputed land at least since January 24, 1947 as required by Sec. 48(b) of the Public Land Act. The basic presumption is that lands of whatever classification belong to the State and evidence of a land grant must be “well-nigh incontrovertible.” As petitioner Abejaron has not adduced any evidence of title to the land in controversy, whether by judicial confirmation of title, or homestead, sale, or free patent, he cannot maintain an action for reconveyance.

OWNERSHIP: Principle of Self-Help

  GERMAN MANAGEMENT & SERVICES, INC. v. COURT OF APPEALS G.R. No. 76217, 14 September 1989

FACTS: The land subject of the case was formerly declared for taxation purposes in the name of Sinforoso Mendoza prior to 1954 but is now declared in the name of Margarito Mendoza. Petitioners are the daughters of Margarito Mendoza while the respondent is the only daughter of Sinforoso Mendoza. Margarito Mendoza and Sinforoso Mendoza were brothers, now deceased. During the cadastral survey of the property on October 15, 1979 there was already a dispute between Honorata M. Bolante and Miguel Mendoza, brother of petitioners. Respondent was occupying the property in question. ISSUE: Whether is or not the doctrine of self help applies in this case RULING: NO. The doctrine of self help, which the petitioners were using to justify their actions, are not applicable in the case because it can only be exercised at the time of actual or threatened dispossession which is absent in the case at bar. Article 536 basically tells us that the owner or a person who has a better right over the land must resort to judicial means to recover the property from another person who possesses the land.

27 4S 2014-2015  

San Beda College of Law CIVIL LAW REVIEW   CAISIP v. PEOPLE G.R. No. L-28716, 18 November 1970 [BONAVENTE] FACTS: Gloria Cabalag is the wife of Marcelino Guevarra who cultivated a parcel of land known as Lot 105-A of Hacienda Palico in Nasugbu, Batangas. Hacienda Palico is owned by Roxas y Cia. The overseer of the said hacienda is Felix Caisip. Roxas y Cia filed an action against Guevarra for forcible entry. The Court ordered Guevarra to vacate the lot and to pay damages and accrued rentals. A writ of execution was issued. Guevarra was given twenty days from June 6, 1959 within which to leave the premises. On June 17, 1959, Gloria Cabalag was seen weeding the portion of Lot 105-A which was a ricefield. Caisip approached her and bade her to leave, but she refused to do so, alleging that she and her husband had the right to stay there and that the crops thereon belong to them. Caisip went to his co-defendants, Sgt. Rojales and Cpl. Villadelrey, both of the local police, who were some distance away, and brought them with him. Rojales told Gloria, who was then in a squatting position, to stop weeding. As Gloria insisted on her right to stay in said lot, Rojales grabbed her right hand and, twisting the same, wrested therefrom the trowel she was holding. Thereupon, Villadelrey held her left hand and, together with Rojales, forcibly dragged her northward towards a forested area, where there was a banana plantation, as Caisip stood nearby, with a drawn gun. Gloria shouted "Ina ko po! Ina ko po!" then her neighbors came and asked the policemen why they were dragging her. The policemen having answered that they would take Gloria to town which was on the west Francisca Andino pleaded that Gloria be released, saying that, if their purpose was as stated by them, she (Gloria) would willingly go with them. By this time, Gloria had already been dragged about eight meters and her dress, as well as her blouse were torn. She then agreed to proceed westward to the municipal building. ISSUE: Were the acts of the appellants justified under Article 429 of the New Civil Code? RULING: NO. Article 429 of the Civil Code upon which appellants rely is obviously inapplicable to the case at bar, for, having been given 20 days from June 6, 1959, within which to vacate Lot 105-A, complainant did not, within said period, invade or usurp said lot. She had merely remained in possession thereof, even though the hacienda owner may have become its co-possessor. Appellants did not "repel or prevent an actual or threatened physical invasion or usurpation." They expelled Gloria from a property of which she and her husband were in possession and despite the fact that the Sheriff had explicitly authorized them to stay in said property up to June 26, 1959. Appellants herein had, by means of violence, and without legal authority therefor, prevented the complainant from "doing something not prohibited by law," (weeding and being in Lot 105-A), and compelled her "to do something against" her will (stopping the weeding and leaving said lot), "whether it be 28 4S 2014-2015  

San Beda College of Law CIVIL LAW REVIEW   right or wrong," thereby taking the law into their hands, in violation of Art. 286 of the Revised Penal Code. Defendants Felix Caisip, Ignacio Rojales and Federico Villadelrey, were convicted of the crime of Grave Coercion aggravated by abuse of superior strength and disregard of the respect due the offended party by reason of her sex. PEOPLE v. PLETCHA, JR. G.R. No. 19029, 27 June 1977 [CABBUAG] FACTS: Tito Pletcha is a farmer who owns a land which he has been cultivating for 19 years. A private corporation sought to take over the aforementioned land by fencing the 4 hectares of his property. Such fencing was without authority or court order. Because of this, Pletcha fought-off any the take over and resisted the company. This forced the company to file a case for grave coercion against Pletcha in the Municipal Court of Murcia, Negros Occidental. Pletcha invokes the protective mantle of Article 429 of the Civil Code which gives him the right to use reasonable force to exclude any person threatening his exclusive ownership over the land. The People asks for affirmance on the ground that the appellant should not have taken the law into his own hands but rather have the courts decide the case. ISSUE: Can Pletcha properly invoked Article 429? RULING: YES. The principle of self-help authorizes the lawful possessor to use force not only to prevent a threatened unlawful invasion or usurpation thereof; it is sort of self-defense. It is lawful to repel force by force. He who merely uses force to defend his possession does not possess by force. The use of such necessary force to protect proprietary or possessory rights constitutes a justifying circumstance under our penal laws.

OWNERSHIP: Use Injuring Rights of Third Persons

  ANDAMO v. INTERMEDIATE APPELLATE COURT and MISSIONARIES OF OUR LADY OF LA SALETTE, INC. G.R. No. 74761, 6 November 1990 [CRON] FACTS: Petitioner spouses Andamo are the owners of a parcel of land situated in Biga (Biluso) Silang, Cavite which is adjacent to that of private respondent, Missionaries of Our Lady of La Salette, Inc., a religious corporation. 29 4S 2014-2015  

San Beda College of Law CIVIL LAW REVIEW   Within the land of respondent corporation, waterpaths and contrivances, including an artificial lake, were constructed, which allegedly inundated and eroded petitioners' land, caused a young man to drown, damaged petitioners' crops and plants, washed away costly fences, endangered the lives of petitioners and their laborers during rainy and stormy seasons, and exposed plants and other improvements to destruction. Petitioners instituted a criminal action for destruction by means of inundation under Article 324 of the Revised Penal Code and a civil action for damages under Articles 2176 and 2177 of the Civil Code. The Court dismissed the civil case in view of the pendency of the criminal action. ISSUE: Can a corporation, which has built through its agents, waterpaths, water conductors and contrivances within its land, thereby causing inundation and damage to an adjacent land, be held civilly liable for damages under Articles 2176 and 2177 of the Civil Code on quasi-delicts such that the resulting civil case can proceed independently of the criminal case? RULING: YES. It must be stressed that the use of one's property is not without limitations. Article 431 of the Civil Code provides that "the owner of a thing cannot make use thereof in such a manner as to injure the rights of a third person." SIC UTERE TUO UT ALIENUM NON LAEDAS. Moreover, adjoining landowners have mutual and reciprocal duties which require that each must use his own land in a reasonable manner so as not to infringe upon the rights and interests of others. Although we recognize the right of an owner to build structures on his land, such structures must be so constructed and maintained using all reasonable care so that they cannot be dangerous to adjoining landowners and can withstand the usual and expected forces of nature. If the structures cause injury or damage to an adjoining landowner or a third person, the latter can claim indemnification for the injury or damage suffered. A quasi-delict or culpa aquiliana is a separate legal institution under the Civil Code with a substantivity all its own, and individuality that is entirely apart and independent from a delict or crime — a distinction exists between the civil liability arising from a crime and the responsibility for quasi-delicts or culpa extra-contractual. The same negligence causing damages may produce civil liability arising from a crime under the Penal Code, or create an action for quasi-delicts or culpa extra-contractual under the Civil Code.

30 4S 2014-2015  

San Beda College of Law CIVIL LAW REVIEW  

OWNERSHIP: Surface Rights

  ANDAMO v. INTERMEDIATE APPELLATE COURT G.R. No. 74761, 6 November 1990 [CAINDAY]

FACTS: Sps. Andamo owned a parcel of land adjacent to the land of the Missionaries of Our Lady of La Salette, Inc. Within the land of the latter, water paths and contrivances, including an artificial lake were constructed, which allegedly inundated and eroded petitioner’s land, caused a young man to drown, damaged petitioner’s crops and plants, washed away costly fences, endangered the lives of petitioners and their laborers and some other destructions. ISSUE: Did respondents improperly exercise their surface right? RULING: YES. Adjoining landowners have mutual and reciprocal duties which require that each must use his own land in a reasonable manner so as not to infringe upon the rights and interests of others. Although we recognize the right of an owner to build structures on his land, such structures must be so constructed and maintained using all reasonable care so that they cannot be dangerous to adjoining landowners and can withstand the usual and expected forces of nature. If the structures cause injury or damage to an adjoining landowner or a third person, the latter can claim indemnification for the injury or damage suffered.

OWNERSHIP: Accession Discreta

  BACHRACH MOTOR CO., INC. v. TALISAY-SILAY CO., INC., ET AL. G.R. No. 35223, 17 September 1931 [DIVINO]

FACTS: On December 22, 1923, the Talisay-Silay Milling Co., Inc., was indebted to the Philippine National Bank. To secure the payment of its debt, it succeeded in inducing its planters, among whom was Mariano Lacson Ledesma, to mortgage their land to the creditor bank. In order to compensate those planters for the risk they were running with their property under the mortgage, the aforesaid central, by a resolution passed on that same date, undertook to credit the owners of the plantation thus mortgaged every year with a sum equal to 2% of the debt secured according to yearly balance. Later on, a complaint was filed by the Bachrach Motor Co., Inc., against the Talisay-Silay Milling Co., Inc., for the delivery of the amount P13,850 as bonus in favor of Mariano Lacson Ledesma. The Philippine National Bank 31 4S 2014-2015  

San Beda College of Law CIVIL LAW REVIEW   filed a third party claim alleging a preferential right to receive any amount which Mariano Lacson Ledesma might be entitled to from the Talisay-Silay Milling Co. as bonus, because that would be civil fruits of the land mortgaged to said bank by said debtor for the benefit of the central referred to. The trial court held that the Bachrach Motor Co., Inc., had a preferred right to receive the amount of P11,076.02 which was Mariano Lacson Ledesma's bonus, and it ordered the defendant central to deliver said sum to the plaintiff. Hence, the appeal of PNB. ISSUE: Is the bonus in question (the compensation for the risk of having subjected one's land to a lien in favor of the bank) considered civil fruits? RULING: NO. Article 355 of the Civil Code considers three things as civil fruits: First, the rents of buildings; second, the proceeds from leases of lands; and, third, the income from perpetual or life annuities, or other similar sources of revenue. It is to be noted that the said bonus bears no immediate, but only a remote accidental relation to the land mentioned, having been granted as compensation for the risk of having subjected one's land to a lien in favor of the bank, for the benefit of the entity granting said bonus. If this bonus be income or civil fruits of anything, it is income arising from said risk, or, if one chooses, from Mariano Lacson Ledesma's generosity in facing the danger for the protection of the central, but certainly it is not civil fruits or income from the mortgaged property, which, as far as this case is concerned, has nothing to do with it. EQUATORIAL REALTY DEVELOPMENT, INC. v. MAYFAIR THEATRE, INC., G.R. NO. 133879, 21 November 2001 [DOMINGO] FACTS: [The case arose out of an earlier case decided by the Court on November 21, 1996.] Carmelo & Bauermann, Inc. used to own a parcel of land, together with two 2storey buildings constructed thereon in Manila. On June 1, 1967, Carmelo entered into a Contract of Lease with Mayfair Theater Inc. for a period of 20 years. The lease covered a portion of the second floor and mezzanine of a two-storey building, wherein the Maxim Theatre was built. Two years later, Mayfair entered into a second Contract of Lease with Carmelo for the lease of another portion of the latter's property. In that space, Mayfair put up another movie house known as Miramar Theater. The Contract of Lease was likewise for a period of 20 years. Both leases contained a provision granting Mayfair a right of first refusal to purchase the subject properties. However, on July 30, 1978 — within the 20-year-lease term — the subject properties were sold by Carmelo to Equatorial Realty Development, Inc. for the total sum of P11,300,000, without their first being offered to Mayfair. As a 32 4S 2014-2015  

San Beda College of Law CIVIL LAW REVIEW   result of the sale of the subject properties to Equatorial, Mayfair filed a Complaint before the Regional Trial Court of Manila for the annulment of the Deed of Absolute Sale between Carmelo and Equatorial, specific performance, and damages. The trial court ruled in favor of Carmelo and Equatorial. On appeal, the decision of the lower court was reversed. On a Petition for Review filed in the SC, the Court rescinded the Deed of Absolute Sale between Carmelo and Mayfair. Such decision became final and executory in 1997. In April 1997, Mayfair filed a Motion for Execution, which the trial court granted. However, Carmelo could no longer be located. Thus, following the order of execution of the trial court, Mayfair deposited with the clerk of court a quo its payment to Carmelo. The lower court issued a Deed of Reconveyance in favor of Carmelo and a Deed of Sale in favor of Mayfair. Meanwhile, in September 1997, Equatorial filed with the RTC of Manila, an action for the collection of a sum of money against Mayfair, claiming payment of rentals or reasonable compensation for the defendant's use of the subject premises after its lease contracts had expired. In its Complaint, Equatorial alleged among other things that the Lease Contract covering the premises occupied by Maxim Theater expired on May 31, 1987, while the Lease Contract covering the premises occupied by Miramar Theater lapsed on March 31, 1989. Representing itself as the owner of the subject premises by reason of the Contract of Sale on July 30, 1978, it claimed rentals arising from Mayfair's occupation thereof. ISSUE: Is Equatorial Realty entitled to payment of back rentals? RULING: NO. Rent is a civil fruit that belongs to the owner of the property producing it by right of accession. Consequently and ordinarily, the rentals that fell due from the time of the perfection of the sale to petitioner until its rescission by final judgment should belong to the owner of the property during that period. In the Law on Sales, ownership of the thing sold is a real right, which the buyer acquires only upon delivery of the thing to him in any of the ways specified in articles 1497 to 1501, or in any other manner signifying an agreement that the possession is transferred from the vendor to the vendee. Delivery may be either actual or constructive, but both forms of delivery contemplate the absolute giving up of the control and custody of the property on the part of the vendor, and the assumption of the same by the vendee. It is an act by which one party parts with the title to and the possession of the property, and the other acquires the right to and the possession of the same. In the case, considering that the petitioner never took actual control and possession of the property sold, in view of respondent's timely objection to the sale and the continued actual possession of the property, it can be said that respondent's opposition to the transfer of the property by way of sale to Equatorial was a legally sufficient impediment that effectively prevented the passing of the property into the latter's hands. The objection took the form of a court action impugning the sale which was rescinded by a judgment rendered by this Court in the mother case. Equatorial as buyer did not acquire a right to 33 4S 2014-2015  

San Beda College of Law CIVIL LAW REVIEW   the fruits of the thing sold from the time the obligation to deliver the property to petitioner arose. Moreover, the Decision in the mother case stated that "Equatorial x x x has received rents" from Mayfair "during all the years that this controversy has been litigated." and the payment thereof of Mayfair to petitioner should not be interpreted to mean either actual delivery or ipso facto recognition of Equatorial's title. The rental payments made by Mayfair should not be construed as a recognition of Equatorial as the new owner. They were made merely to avoid imminent eviction.

OWNERSHIP: Accession Continua

  DAMIAN IGNACIO ET AL v. ELIAS HILARIO, ET AL. G.R. No. L-175, 30 April 1946 [ESTEBAN]

FACTS: Elias Hilario and his wife Dionisia Dres filed a case before the CFI of Pangasinan against Damian, Francisco and Luis, surnamed Ignacio, concerning the ownership of a parcel of land, partly rice-land and partly residential. After the trial of the case, the lower court, presided over by Hon. Alfonso Felix, rendered judgment holding plaintiffs as the legal owners of the whole property but conceding to defendants the ownership of the houses and granaries built by them on the residential portion with the rights of a possessor in good faith, in accordance with Article 361 of the Civil Code. Subsequently, in a motion filed in the same Court of First Instance the plaintiffs prayed for an order of execution alleging that since they chose neither to pay defendants for the buildings nor to sell to them the residential lot, said defendants should be ordered to remove the structure at their own expense and to restore plaintiffs in the possession of said lot. ISSUE: May an owner in good faith eject a builder in good faith without choosing either to appropriate the building for himself after payment of its value or to sell his land to the builder in good faith? RULING: NO. The owner of the building erected in good faith on a land owned by another, is entitled to retain the possession of the land until he is paid the value of his building, under Article 453. The owner of the land, upon the other hand, has the option, under Article 361, either to pay for the building or to sell his land to the owner of the building. But he cannot, as respondents here did, refuse both to pay for the building and to sell the land and compel the owner of the building to remove it from the land where it is erected. He is entitled to such remotion only when, after having chosen to sell his land, the other party fails to pay for the same. 34 4S 2014-2015  

San Beda College of Law CIVIL LAW REVIEW   FLORENCIO IGNAO v. HON. INTERMEDIATE APPELLATE COURT, JUAN IGNAO, substituted by his Legal Heirs, and ISIDRO IGNAO G.R. No. 72876, 18 January 1991 [FERNANDEZ] FACTS: Petitioner Florencio Ignao and his uncles private respondents Juan Ignao and Isidro Ignao were co-owners of a parcel of land with an area of 534 square meters situated in Barrio Tabon, Kawit, Cavite. Pursuant to an action for partition, the then CFI of Cavite directed the partition of the aforesaid land, alloting 133.5 square meters to private respondents Juan and Isidro, and giving the remaining portion with a total area of 266.5 square meters to petitioner Florencio. However, no actual partition was ever effected. Petitioner instituted a complaint for recovery of possession of real property against private respondents alleging that the area occupied by the two (2) houses built by private respondents exceeded the 133.5 square meters previously alloted to them. The lower court found that the houses of Juan and Isidro actually encroached upon a portion of the land belonging to Florencio. The CFI ruled that although private respondents occupied a portion of Florencio's property, they should be considered builders in good faith and pursuant to Article 448 of the Civil Code, the owner of the land (Florencio) should have the choice to either appropriate that part of the house standing on his land after payment of indemnity or oblige the builders in good faith (Juan and Isidro) to pay the price of the land. However, the trial court observed that based on the facts of the case, it would be useless and unsuitable for Florencio to exercise the first option since this would render the entire houses of Juan and Isidro worthless. It ordered the plaintiff Florencio Ignao to sell to the defendants Juan and Isidro Ignao that portion of his property with an area of 101 square meters at P40.00 per square meter, on which part the defendants had built their houses. On appeal, the IAC affirmed the CFI’s decision. ISSUES: 1. Should the provisions of Article 448 apply to a builder in good faith on a property held in common? 2. May the court choose from the options provided in Article 448 for the petitioner? RULING: 1. YES. When the co-ownership is terminated by a partition and it appears that the house of an erstwhile co-owner has encroached upon a portion pertaining to another co-owner which was however made in good faith, then the provisions of Article 448 should apply to determine the respective rights of the parties.

35 4S 2014-2015  

San Beda College of Law CIVIL LAW REVIEW   2. NO. In Quemuel v. Olaes, the Court categorically ruled that the right to appropriate the works or improvements or to oblige the builder to pay the price of the land belongs to the landowner. FILIPINAS COLLEGES, INC. v. MARIA GARCIA TIMBANG, ET AL. G.R. No. L-12812, 29 September 1959 [J.FERNANDO] FACTS: The parties in this case are the spouses Timbang, the Filipinas Colleges, Inc., and Maria Gervacio Blas. Blas was declared to be a builder in good faith for the school building constructed on the subject lot and entitled to be paid the amount of the building. While Filipinas Colleges, Inc. was declared to have acquired the rights of the spouses Timbang in the lot subject to the payment of the value of the land. In case Filipinas Colleges, Inc. failed to deposit the value of the land within the time fixed by the court, Filipinas Colleges would lose all its rights to the land and the spouses Timbang would then become the owners thereof. In that eventuality, the Timbangs would make known to the court their option under Art. 448 of the Civil Code whether they would appropriate the building in question, in which even they would have to pay Filipinas Colleges, Inc. the sum of P19,000.00, or would compel the latter to acquire the land and pay the price thereof. It is contended by the appellants that because the builder in good faith has failed to pay the price of the land after the owners thereof exercised their option under Article 448 of the Civil Code, the builder lost his right of retention provided in Article 546 and by operation of Article 445, the appellants as owners of the land automatically became the owners ipso facto, the execution sale of the house in their favor was superfluous. ISSUE: Is the failure of the builder to pay the value of the land, when such is demanded by the land-owner, the latter becomes automatically the owner of the improvement? RULING: NO. Under the terms of articles 448 and 546, it is true that the owner of the land has the right to choose between appropriating the building by reimbursing the builder of the value thereof or compelling the builder in good faith to pay for his land. Even this second right cannot be exercised if the value of the land is considerably more than that of the building. In addition to the right of the builder to be paid the value of his improvement, Article 546 gives him the corollary right of retention of the property until he is indemnified by the owner of the land. There is nothing in the language of these two article, 448 and 546, which would justify the conclusion of appellants that, upon the failure of the builder to pay the value of the land, when such is demanded by the land-owner, the latter becomes automatically the owner of the improvement under Article 445. 36 4S 2014-2015  

San Beda College of Law CIVIL LAW REVIEW   MANOTOK REALTY INC. v. TECSON G.R. No. L-47475, 19 August 1988 [L. FERNANDO] FACTS: In a complaint filed by the petitioner Manotok Realty for recovery of possession of land against private respondent Nilo Madlangawa, CFI ruled declaring the said respondent a builder in good faith. CA affirmed and SC dismissed for lack of merit. Petitioner then filed with the trial court motion for the approval of the petitioner's exercise of option to appropriate the improvements introduced by the private respondent on the property under Article 448 of the Civil Code, and thereafter, for satisfaction of judgment(that is final and executory) by ordering respondents to deliver possession of the property in question to the petitioner. The motion was dismissed. Hence this petition for mandamus alleging that the respondent judge committed grave abuse of discretion in denying petitioner’s motion on the grounds that under Articles 448 and 546 of the Civil Code, the exercise of option belongs to the owner of the property, who is the petitioner herein, and that upon finality of judgment, the prevailing party is entitled, as a matter of right, to its execution which is only a ministerial act on the part of the respondent judge.Petitioner also claimed that since the house of the private respondent (which was the improvement built on the land owned by petitioner) was gutted by fire, the execution of the decision would now involve the delivery of possession of the disputed area by the private respondent to the petitioner. ISSUE: Should the court Issue the writ of execution for the delivery of possession by the builder in good faith of the subject land to the owner thereof after a fire gutted the improvements built by the former over the said land which serves as basis of his right to retain the same? RULING: YES. Where the improvements have been destroyed by a fortuitous event without the fault of the landowner, the basis for the builder’s right to retain the premises is extinguished; hence there is no other recourse for him but to vacate the premises and deliver the same to the landowner. Moreover, even without the fire, the answer will still be the same. The option given by law under Article 448 of the Civil Code, i.e., either to retain the premises and pay for the improvements thereon or to sell the said premises to the builder in good faith belongs to the owner of the property. The only right given to the builder in good faith is the right to reimbursement for the improvements; the builder cannot compel the owner of the land to sell such land to the former. THEREFORE, the judge cannot deny the issuance of a writ of execution for the delivery of possession of the subject land, had the owner chose to retain the premises and pay for the improvements thereon, because the other party was adjudged a builder in good faith or on the ground of “peculiar circumstances which supervened after the institution of this case. 37 4S 2014-2015  

San Beda College of Law CIVIL LAW REVIEW   BERNARDO v. BATACLAN G.R. No. L-44606, 28 November 1938 [FLORES] FACTS: Plaintiff Vicente Bernardo acquired a parcel of land from Pastor Samonte through a contract of sale. Thereafter, Bernardo instituted a case against said Vendor to secure a possession of the land. Bernardo was able to obtain a favorable decision from the court. He then found defendant Catalino Bataclan in the said premises. It appears that he has been authorized by the former owners, as far back as 1922, to clear the land and make improvements thereon. Thus, plaintiff instituted a case against Bataclan in the Court of First Instance of Cavite. In this case, plaintiff was declared the owner of the land but the defendant was held to be a possessor in good faith, entitled to reimbursement in the total sum of P1,646, for work done and improvements made. On appeal, the court made some modifications by allowing the defendant to recover compensation amounting to P2,212 and by reducing the price at which the plaintiff could require the defendant to purchase the land in question from P300 down to P200 per hectare. Plaintiff conveyed to the court his desire to require the defendant to pay the land at the rate of P200 per hectare or a total price of P18,000 for the whole tract of land. Defendant failed to pay the land. ISSUE: Can Bataclan still exercise his right of retention? RULING: NO. The Civil Code confirms time-honored principles of the law of property. One of these is the principle of accession whereby the owner of property acquires not only that which it produces but that which is united to it either naturally or artificially (Article 353). Whatever is built, planted or sown on the land of another, and the improvements on repairs made thereon, belong to the owner of the land (Article 358). Where, however, the planter, builder, or sower has acted in good faith, a conflict of rights arises between the owners and it becomes necessary to protect the owner of the improvements without causing injustice to the owner of the land. In view of the impracticability of creating what Manresa calls a state of “forced ownership”, the law has provided a just and equitable solution by giving the owner of the land the option to acquire the improvements after payment of the proper indemnity or to oblige the builder or planter to pay for the land and the sower to pay the proper rent (Article 361). It is the owner of the land who is allowed to exercise the option because his right is older and because, by the principle of accession, he is entitled to ownership of the accessory thing. The law requires no more than that the owner of the land should choose between indemnifying the owner of the improvements or requiring the latter to pay for the land. When defendant Bataclan failed to pay the land, he herein lost his right of retention. 38 4S 2014-2015  

San Beda College of Law CIVIL LAW REVIEW   HEIRS OF RAMON DURANO, SR. v. SPOUSES UY G.R. No. 136456, 24 October 2000 [JAVIER] FACTS: Petitioners instituted an action for damages against the respondents because of allegedly officiating a “hate campaign” against them over petitioners’ socalled “invasion” of respondents’ alleged properties. Even before many of the respondents received notices to vacate, men who identified themselves as employees of Durano & Co. proceeded to bulldoze the lands occupied by various respondents, destroying in their wake the plantings and improvements made by the respondents therein. In their counterclaim, respondents alleged that petitioners’ acts deprived most of them of their independent source of income and have made destitutes of some of them. Also, petitioners have done serious violence to respondents’ spirit, as citizens and human beings, to the extent that one of them had been widowed by the emotional shock that the damage and dispossession has caused. Thus, in addition to the dismissal of the complaint, respondents demanded actual damages for the cost of the improvements they made on the land, together with the damage arising from the dispossession itself; moral damages for the anguish they underwent as a result of the high-handed display of power by petitioners in depriving them of their possession and property; as well as exemplary damages, attorney’s fees and expenses of litigation. The court found that the properties properly belonged to the respondents by way of purchase or inheritance and that the titles procured in the name of Ramon Durano III were attended by fraud. ISSUE: What is the basis of damages to be paid by a builder in bad faith under Art. 451 of the Civil Code?

RULING: The right of the owner of the land to recover damages from a builder in bad faith is clearly provided for in Article 451 of the Civil Code. Although said Article 451 does not elaborate on the basis for damages, the Court perceives that it should reasonably correspond with the value of the properties lost or destroyed as a result of the occupation in bad faith, as well as the fruits (natural, industrial or civil) from those properties that the owner of the land reasonably expected to obtain.

The owner of the land has three alternative rights: (1) to appropriate what has been built without any obligation to pay indemnity therefor, or (2) to demand that the builder remove what he had built, or (3) to compel the builder to pay the value of the land. In any case, the landowner is entitled to damages under Article 451. 39 4S 2014-2015  

San Beda College of Law CIVIL LAW REVIEW   BALLATAN v. COURT OF APPEALS G.R. No. 125683, 2 March 1999 [KING] FACTS: The instant case arose from a dispute over forty-two (42) square meters of residential land belonging to petitioners. In 1985, petitioner Ballatan constructed her house on Lot No. 24. During the construction, she noticed that the concrete fence and side pathway of the adjoining house of respondent Winston Go encroached on the entire length of the eastern side of her property. Her building contractor informed her that the area of her lot was actually less than that described in the title. Forthwith, Ballatan informed respondent Go of this discrepancy and his encroachment on her property. Respondent Go, however, claimed that his house, including its fence and pathway, were built within the parameters of his father's lot; and that this lot was surveyed by Engineer Jose Quedding, the authorized surveyor of the Araneta Institute of Agriculture (AIA), the owner-developer of the subdivision project. Petitioner Ballatan called the attention of the AIA to the discrepancy of the land area in her title and the actual land area received from them. The AIA authorized another survey of the land by Engineer Jose N. Quedding. In a report dated February 28, 1985, Engineer Quedding found that the lot area of petitioner Ballatan was less by a few meters and that of respondent Li Ching Yao, which was three lots away, increased by two (2) meters. In short, Lots Nos. 25, 26 and 27 moved westward to the eastern boundary of Lot No. 24. ISSUES: 1. Are respondents builders in good faith? 2. What are the rights of petitioners on said dispute? RULING: 1. YES. The appellate court, however, found that it was the erroneous survey by Engineer Quedding that triggered these discrepancies. Respondents Go are deemed builders in good faith until the time petitioner Ballatan informed them of their encroachment on her property. Furthermore, there is no evidence, much less, any allegation that respondent Li Ching Yao was aware that when he built his house he knew that a portion thereof encroached on respondents Go's adjoining land. Good faith is always presumed, and upon him who alleges bad faith on the part of a possessor rests the burden of proof. All the parties are presumed to have acted in good faith.

40 4S 2014-2015  

San Beda College of Law CIVIL LAW REVIEW   2. Petitioners, as owners of Lot No. 24, may choose to purchase the improvement made by respondents Go on their land, or sell to respondents Go the subject portion. If buying the improvement is impractical as it may render the Go's house useless, then petitioners may sell to respondents Go that portion of Lot No. 24 on which their improvement stands. If the Go's are unwilling or unable to buy the lot, then they must vacate the land and, until they vacate, they must pay rent to petitioners. Petitioners, however, cannot compel respondents Go to buy the land if its value is considerably more than the portion of their house constructed thereon. If the value of the land is much more than the Go's improvement, then respondents Go must pay reasonable rent. If they do not agree on the terms of the lease, then they may go to court to fix the same. In the event that petitioners elect to sell to respondents Go the subject portion of their lot, the price must be fixed at the prevailing market value at the time of payment. SPOUSES DEL CAMPO v. ABESIA G.R. No.L-49219, 15 April 1988 [LIBOON] FACTS: An action for partition was filed by plaintiffs in the CFI of Cebu. Plaintiffs and defendants are co-owners pro indiviso of this lot in the proportion of and 1/3 share each, respectively. The trial court appointed a commissioner who recommended that the lot be divided into two portion with the area of 30 sqm for the plaintiffs and 15 sqm for the defendants. The houses of plaintiffs and defendants were surveyed and shown on the sketch plan. The house of defendants occupied the portion with an area of 5 square meters of Lot 1161A of plaintiffs. The parties manifested their conformity to the report and asked the trial court to finally settle and adjudicate who among the parties should take possession of the 5 square meters of the land in question. ISSUE: Is Art 448 of the New Civil Code applicable in cases where the disputed land is co-owned? RULING: NO. Article 448 of the New Civil Code provides as follows: Art. 448. The owner of the land on which anything has been built, sown, or planted in good faith, shall have the right to appropriate as his own the works, sowing or planting, after payment of the indemnity provided for in articles 546 and 548, or to oblige the one who built or planted to pay the price of the land, and the one who sowed, the proper rent. However, the builder or planter cannot be obliged to buy the land if its value is considerably more than that of the building or trees. In such case, he shall pay reasonable rent, if the owner of the land does not choose to appropriate the building or trees after proper indemnity. The parties shall agree upon the terms of the lease and in case of disagreement, the court shall fix the terms thereof.

The court a quo correctly held that Article 448 of the Civil Code cannot apply where a co-owner builds, plants or sows on the land owned in common for 41 4S 2014-2015  

San Beda College of Law CIVIL LAW REVIEW   then he did not build, plant or sow upon land that exclusively belongs to another but of which he is a co-owner. The co-owner is not a third person under the circumstances, and the situation is governed by the rules of coownership. However, when, as in this case, the co-ownership is terminated by the partition and it appears that the house of defendants overlaps or occupies a portion of 5 square meters of the land pertaining to plaintiffs which the defendants obviously built in good faith, then the provisions of Article 448 of the new Civil Code should apply. Manresa and Navarro Amandi agree that the said provision of the Civil Code may apply even when there was co-ownership if good faith has been established. PACIFIC FORMS, INC. v. ESGUERRA G.R. No. L-21783, 29 November 1969 [LIM] FACTS: Defendant –appellant, Carried Lumber Co. delivered and sold construction materials to Insular Farms, Inc. such was utilized for construction of six buildings at its compound in Bolinao Pangasinan within the period of Oct.1,1956- Mar.2, 1957. The total procurement amounted to P15, 000.00 but Insular Farms, Inc paid only partial amount of P4, 710.18. Carried Lumber, Co instituted a civil case to recover the unpaid balance before the CFI of Pangasinan which sustained the company’s claim, writ of execution was issued. The sheriff levied the six buildings. The plaintiff- appellee, Pacific Farms, Inc, filed a third party complaint asserting absolute ownership of the buildings it was sold by Insular Farms seven prior to the civil case. They seek to nullify the levy and judicial sale with damages. The corporation contended to be innocent buyer in good faith for value. Hence, this appeal. ISSUE: Is there a materialman's lien over the six buildings in favor of the appellant to be paid by the buyer of property? RULING: YES. Therefore, applying article 447 by analogy, we perforce consider the buildings as the principal and the lumber and construction materials that went into their construction as the accessory. Thus the appellee, if it does own the six buildings, must bear the obligation to pay for the value of the said materials; the appellant — which apparently has no desire to remove the materials, and, even if it were minded to do so, cannot remove them without necessarily damaging the buildings — has the corresponding right to recover the value of the unpaid lumber and construction materials. 42 4S 2014-2015  

San Beda College of Law CIVIL LAW REVIEW   The court noted evident circumstances that negate the appellee's claim of being innocent buyer in good faith for value: · In the deed of absolute sale, exhibit 1, the Insular Farms, Inc. (vendor) was represented in the contract by its president, J. Antonio Araneta. The latter was a director of the appellee (Pacific Farms, Inc.) and was the counsel who signed the complaint filed by the appellee in the court below. J. Antonio Araneta was, therefore, not only the president of the Insular Farms, Inc. but also a director and counsel of the appellee.

·

During the trial of civil case the Insular Farms, Inc. was represented by Attorney Amado Santiago, Jr. of the law firm of J. Antonio Araneta. The latter was one of the counsels of the Pacific Farms, Inc.

PECSON v. COURT OF APPEALS G.R. No. 115814, 26 May 1995 [MADRILENO] FACTS: Petitioner Pedro P. Pecson was the owner of a commercial lot located in Kamias Street, Quezon City, on which he built a four-door two-storey apartment building. For his failure to pay realty taxes amounting to twelve thousand pesos (P12,000.00), the lot was sold at public auction by the city Treasurer of Quezon City to Mamerto Nepomuceno who in turn sold it on 12 October 1983 to the private respondents, the spouses Juan Nuguid and Erlinda Tan-Nuguid, for one hundred three thousand pesos (P103,000.00). The petitioner challenged the validity of the auction sale in Civil Case No. Q41470 before the RTC of Quezon City. In its decision of 8 February 1989, the RTC dismissed the complaint, but as to the private respondents' claim that the sale included the apartment building, it held that the issue concerning it was "not a subject of the litigation." In resolving the private respondents' motion to reconsider this issue, the trial court held that there was no legal basis for the contention that the apartment building was included in the sale. After an entry of judgment was made, the Sps. Nuguid filed a motion with the RTC for a motion for delivery of possession of the lot and the apartment bldg citing Art. 546 of the CC. The RTC issued an order declaring that the owner of the lot and apartment bldg were the Sps. Nuguid and to pay the construction cost of the apartment before a writ of possession would be issued and to pay rent to the spouses. Pecson moved for reconsideration but the Trial court did not act on it, instead it issued a writ of possession. The CA affirmed in part the decision declaring the cost of construction can be offset from the amount of rents to be collected and that since Sps. Nuguid opted to appropriate the improvement, Pecson is entitled to be reimbursed the cost of construction at the time it was built in 1965 which is at P53k and the right the retain the improvement until full indemnity is paid. ISSUE: Does Art. 448 and 546 apply in the case at bar RULING: YES. With regard to Art. 448, the provision on indemnity may be applied in analogy. Whoever is the owner of the land may appropriate whatever has 43 4S 2014-2015  

San Beda College of Law CIVIL LAW REVIEW   been built, planted or sown after paying indemnity. However, it does not apply when the owner of the land is also the builder of the works on his own land who later on loses ownership by sale or donation. On the other hand, Art. 546 refers to the necessary and useful expenses which shall be refunded to the possessor in good faith with right of retention. However, it does not state how to determine the value of the useful improvement. The respondent court and private respondents alike claims that the construction cost in 1965 constitute sufficient reimbursement, however, this is contrary to previous rulings which declares that the value to the reimbursed should be the present market value of said improvements so as not to unjustly enrich either of the parties. The trial court erred in ordering Pecson to pay rent since the Sps. Nuguid has yet to pay the indemnity, therefore Pecson has the right to retain the improvements and the income thereof. The case was remanded to the trial court for determination of the current market value of the apartment bldg and ordered the Sps. to pay Pecson otherwise it shall be restored to Pecson until payment of indemnity.

OWNERSHIP: Good Faith

  TECHNOGAS PHILIPPINES MANUFACTURING COURT OF APPEALS AND EDUARDO UY G.R. No. 108894, 10 February 1997 [MAGULTA]

CORPORATION

v.

FACTS: Petitioner, Technogas Philippines Manufacturing Corporation (Technogas) is the registered owner of a parcel of land in Paranaque, Metro Manila. The said land was purchased by plaintiff from Pariz Industries Inc. in 1970, together with all the buildings and improvements including the wall existing thereon. On the other hand, respondent Eduardo Uy is the registered owner of a land adjoining that of petitioner. It was discovered in a survey that portions of the buildings and wall bought by plaintiff together with the land from Pariz Industries are occupying a portion of defendant’s adjoining land. Upon learning of the enroachment, plaintiff offered to buy from defendant that particular portion occupied by portions of its buildings and wall. Defendant, however, refused. In 1973, the parties entered into a private agreement wherein plaintiff agreed to demolish the wall, thus giving the defendant possession of a portion of his land previously enclosed by plaintiff’s wall. Uy later files a complaint before the Municipal Engineer of Paranaque as well as before the Office of the Provincial Fiscal against Technogas but the complaint did not prosper. Uy thereafter dug or caused to be dug a cannal along Techogas’ wall, a portion of which collapsed. This led to the filing by the petitioner of a supplemental complaint in the above mentioned case and a separate criminal complaint for malicious mischief against Uy and his wife. While trial of the case was in progress, plaintiff filed in 44 4S 2014-2015  

San Beda College of Law CIVIL LAW REVIEW   Court a formal proposal for settlement of the case but said proposal, however, was ignored by defendant. The RTC ruled in favor of Technogas ordering Uy to sell to Technogas the portion of land enroached by the building. The CA reversed the decision of the RTC ruling that petitioner is a builder in bad faith because he is presumed to know the metes and bounds of his property. ISSUE: Is bad faith imputable to a registered owner of a land when a part of his building enroached upon a neighbor’s land, simply because he is supposedly presumed to know the boundaries of his land as described in his certificate of title RULING: NO. Unless one is versed in the science of surveying, “no one can determine the precise extent or location of his property by merely examining his paper title.” Article 527 of the Civil Code presumes good faith, and since no proof exists to show that the encroachment over a narrow, needle-shaped portion of private respondent's land was done in bad faith by the builder of the encroaching structures, the latter should be presumed to have built them in good faith. It is presumed that possession continues to be enjoyed in the same character in which it was acquired, until the contrary is proved. Good faith consists in the belief of the builder that the land he is building on is his, and his ignorance of any defect or flaw in his title. Further, "(w)here one derives title to property from another, the act, declaration, or omission of the latter, while holding the title, in relation to the property, is evidence against the former. And possession acquired in good faith does not lose this character except in case and from the moment facts exist which show that the possessor is not unaware that he possesses the thing improperly or wrongfully. The good faith ceases from the moment defects in the title are made known to the possessor, by extraneous evidence or by suit for recovery of the property by the true owner. As to the question whether the same benefit can be invoked by petitioner is not the builder of the offending structures bus possess them as buyer, the court answered in the affirmative. There is no sufficient showing that petitioner was aware of the encroachment at the time it acquired the property from Pariz Industries. In any case, contrary proof has not overthrown the presumption of good faith under Article 527 of the Civil Code, as already stated, taken together with the disputable presumptions of the law on evidence. In the second place, upon delivery of the property by Pariz Industries, as seller, to the petitioner, as buyer, the latter acquired ownership of the property. Consequently, petitioner is deemed to have stepped into the shoes of the seller in regard to all rights of ownership over the immovable sold, including the right to compel the private respondent to exercise either of the two options provided under Article 448 of the Civil Code.

45 4S 2014-2015  

San Beda College of Law CIVIL LAW REVIEW   PLEASANTVILLE DEVELOPMENT APPEALS G.R. No. 79688, 1 February 1996 [MANRIQUE]

CORPORATION

v.

COURT

OF

FACTS: Edith Robillo purchased from petitioner a parcel of land designated as Lot 9 in Pleasantville Subdivision. In 1975, respondent Eldred Jardinico bought the rights to the lot from Robillo. At that time, Lot 9 was vacant. Upon completing all payments, Jardinico secured from the Register of Deeds a Transfer Certificate of Title in his name. It was then that he discovered that improvements had been introduced on Lot 9 by respondent Wilson Kee, who had taken possession thereof. It appears that on March 26, 1974, Kee bought on installment Lot 8 of the same subdivision from C.T. Torres Enterprises, Inc. (CTTEI), the exclusive real estate agent of petitioner. Under the Contract to Sell on Installment, Kee could possess the lot even before the completion of all installment payments. On January 20, 1975, Kee paid CTTEI the relocation fee of P50.00 and another P50.00 on January 27, 1975, for the preparation of the lot plan. These amounts were paid prior to Kee’s taking actual possession of Lot 8. After the preparation of the lot plan and a copy thereof given to Kee, CTTEI through its employee, Zenaida Octaviano, accompanied Kee’s wife, Donabelle Kee, to inspect Lot 8. Unfortunately, the parcel of land pointed by Octaviano was Lot 9. Thereafter, Kee proceeded to construct his residence, a store, an auto repair shop and other improvements on the lot. After discovering that Lot 9 was occupied by Kee, Jardinico confronted him. The parties tried to reach an amicable settlement, but failed. On January 30, 1981, Jardinico’s lawyer wrote Kee, demanding that the latter remove all improvements and vacate Lot 9. When Kee refused to vacate, Jardinico filed a complaint for ejectment with damages against Kee. Kee, in turn, filed a third-party complaint against petitioner and CTTEI. ISSUE: Was the respondent a builder in good faith? RULING: YES. Good faith consists in the belief of the builder that the land he is building on is his and his ignorance of any defect or flaw in his title. And as good faith is presumed, petitioner has the burden of proving bad faith on the part of the respondent (Kee). At the time he built improvements on Lot 8, Kee believed that said lot was what he bought from petitioner. He was not aware that the lot delivered to him was not Lot 8. Thus, Kee's good faith. Petitioner failed to prove otherwise.

46 4S 2014-2015  

San Beda College of Law CIVIL LAW REVIEW   GEMINIANO, ET. AL. v. COURT OF APPEALS G.R. No. 120303, 24 July 1996 [MATEO] FACTS: Paulina Amado vda. de Geminiano, petitioners’ mother, owns a with an area of 314 square meters. On a 12-square-meter portion of that lot stood the petitioners' unfinished bungalow, which the petitioners sold in November 1978 to the private respondents for the sum of P6,000.00, with an alleged promise to sell to the latter that portion of the lot occupied by the house. Subsequently, the petitioners' mother executed a contract of lease over a 126 square-meter portion of the lot, including that portion on which the house stood, in favor of the private respondents for P40.00 per month for a period of seven years commencing on 15 November 1978. The private respondents then introduced additional improvements and registered the house in their names. After the expiration of the lease in November 1985, however, the petitioners' mother refused to accept the monthly rentals.It turned out that the lot in question was the subject of a suit, which resulted in its acquisition by one Maria Lee in 1972. In 1982, Lee sold the lot to Lily Salcedo, who in turn sold it in 1984 to the spouses Agustin and Ester Dionisio. RTC ruled that since the private respondents were assured by the petitioners that the lot they leased would eventually be sold to them, they could be considered builders in good faith, and as such, were entitled to reimbursement of the value of the house and improvements with the right of retention until reimbursement had been made. ISSUE: Are the private respondents builders in good faith? RULING: NO. Being mere lessees, the private respondents knew that their occupation of the premises would continue only for the life of the lease. Plainly, they cannot be considered as possessors nor builders in good faith. This Court has held that Article 448 of the Civil Code, in relation to Article 546 of the same Code, which allows full reimbursement of useful improvements and retention of the premises until reimbursement is made, applies only to a possessor in good faith, i.e., one who builds on land with the belief that he is the owner thereof. It does not apply where one's only interest is that of a lessee under a rental contract; otherwise, it would always be in the power of the tenant to "improve" his landlord out of his property. And even if the petitioners indeed promised to sell, it would not make the private respondents possessors or builders in good faith so as to be covered by the provisions of Article 448 of the Civil Code. The latter cannot raise the mere expectancy of ownership of the aforementioned lot because the alleged promise to sell was not fulfilled nor its existence even proven. The first thing that the private respondents should have done was to reduce the alleged promise into writing, because under Article 1403 of the Civil Code, an agreement for the sale of real property or an interest therein is unenforceable, unless some note or memorandum thereof be produced. Not having taken any steps in order that the alleged promise to sell may be enforced, the private respondents cannot 47 4S 2014-2015  

San Beda College of Law CIVIL LAW REVIEW   bank on that promise and profess any claim nor color of title over the lot in question.

OWNERSHIP: Accession Natural; Alluvium or Alluvion

  AGUSTIN v. INTERMEDIATE APPELLATE COURT G.R. Nos. L-66075-76, 5 July 1990 [MEDINA]

FACTS: The Cagayan River separates the towns of Solana on the west and Tuguegarao on the east in the province of Cagayan. The land east of the Cagayan River is owned by defendant-petitioner Eulogio Agustin (Tuguegarao Cadastre). As the years went by, the Cagayan River moved gradually eastward, depositing silt on the western bank. The shifting of the river and the siltation started by 1919 and continued until 1968. Through the years, the Cagayan River eroded lands of the Tuguegarao Cadastre on its eastern bank among which was defendant-petitioner Eulogio Agustin's, depositing the alluvium as accretion on the land possessed by Pablo Binayug on the western bank. However, in 1968, after a big flood, the Cagayan River changed its course, returned to its 1919 bed, and, in the process, cut across the lands of Maria Melad, Timoteo Melad, and the spouses Pablo Binayug and Geronima Ubina whose lands were transferred on the eastern, or Tuguegarao, side of the river. To cultivate those lots they had to cross the river. In April, 1969, while the private respondents and their tenants were planting corn on their lots located on the eastern side of the Cagayan River, the petitioners, accompanied by the mayor and some policemen of Tuguegarao, claimed the same lands as their own and drove away the private respondents from the premises. On April 21, 1970, private respondents filed a complaint to recover their lots and their accretions. ISSUE: Are private respondents entitled to recover their lots and their accretions? RULING: YES. There had been accretions to the lots of the private respondents and they did not lose the ownership of such accretions even after they were separated from the principal lots by the sudden change of course of the river. Art. 457 of the New Civil Code which provides that to the owners of lands adjoining the banks of rivers belong the accretion which they gradually receive from the effects of the current of the waters. 48 4S 2014-2015  

San Beda College of Law CIVIL LAW REVIEW   Accretion benefits a riparian owner when the following requisites are present: (1) that the deposit be gradual and imperceptible; (2) that it resulted from the effects of the current of the water; and (3) that the land where accretion takes place is adjacent to the bank of a river. All these requisites of accretion are present in this case for the accretion on the western bank of the Cagayan River had been going on from 1919 up to 1968 or for a period of 49 years. It was gradual and imperceptible. The private respondents' ownership of the accretion to their lands was not lost upon the sudden and abrupt change of the course of the Cagayan River in 1968 or 1969 when it reverted to its old 1919 bed, and separated or transferred said accretions to the other side (or eastern bank) of the river. Articles 459 and 463 of the New Civil Code apply to this situation. Art. 459. Whenever the current of a river, creek or torrent segregates from an estate on its bank a known portion of land and transfers it to another estate, the owner of the land to which the segregated portion belonged retains the ownership of it, provided that he removes the same within two years. Art. 463. Whenever the current of a river divides itself into branches, leaving a piece of land or part thereof isolated, the owner of the land retains his ownership. He also retains it if a portion of land is separated from the estate by the current. (Emphasis supplied).

In the case at bar, the sudden change of course of the Cagayan River as a result of a strong typhoon in 1968 caused a portion of the lands of the private respondents to be "separated from the estate by the current." The private respondents have retained the ownership of the portion that was transferred by avulsion to the other side of the river. CUREG v. INTERMEDIATE APPELLATE COURT G.R. No. 73465, 7 September 1989 [MENESES] FACTS: Private respondent Domingo Apostol bought a parcel of land from private respondents Soledad Gerardo, Rosa Gerardo, Nieves Gerardo, Flordeliza Gerardo (Gerardos), and Maquinad. At the time of the execution by the vendors of an Extra-Judicial Partition of the property, the parcel of land already showed signs of accretion of about three (3) hectares. Thereafter, when private respondents were about to cultivate the land together with its accretion, they were prevented and threatened by petitioner Cureg. Petitioner Cureg anchors his claim on the parcel of land and its accretion on Original Certificate of Title No. P-19093 (the OCT only covered the parcel of land), while private respondents anchor their claim on previously filed tax declarations stating the area and boundaries of the land. [In sum, the Court ruled in favor of Cureg, the latter being the rightful owner of the parcel of land as well as the accretion. Other facts are not anymore relevant to the issue that is related to Property. Actually, the property-related issue in this case was only a minor one, was tackled by the SC at the end of the decision, and only in passing].

49 4S 2014-2015  

San Beda College of Law CIVIL LAW REVIEW   ISSUE: Is Cureg still required to register, under the Torrens System, the land acquired through accretion, even if the parcel of land which received the accretion was already covered by a Torrens title? RULING: YES. An increase in the area of one’s land through an accretion left by the change of course or the northward movement of a body of water does not automatically become registered land just because the lot which receives such accretion is covered by a Torrens title. As such, it must also be placed under the operation of the Torrens System. VIAJAR v. COURT OF APPEALS G.R. No. L-77294, December 12, 1988 [ORSUA] FACTS: The spouses Ladrido were the owners of Lot No. 7511 of the Cadastral Survey situated in barangay Cawayan, Pototan, Iloilo. This lot was registered in the names of the spouses under TCT of the Register of Deeds of Iloilo. Spouses Te were also the registered owners of a parcel of land described in their title as Lot No. 7340 of the Cadastral Survey. Te sold this lot Spouses Viajar. A Torrens title was later issued in their names. Later, Angelica Viajar had Lot No. 7340 relocated and found out that the property was in the possession of Ricardo Ladrido. Consequently, she demanded its return but Ladrido refused. Spouses Viajar instituted a civil action for recovery of possession and damages against Ricardo Ladrido and Rosendo Te. Plaintiffs sought the annulment of the deed of sale and the restitution of the purchase price with interest in the event the possession of defendant Ladrido is sustained. It is admitted by the parties that Lot No. 7511 and Lot No. 7340 were separated by the Suague River. Petitioners also contend that Article 457 of the New Civil Code must be construed to limit the accretion mentioned therein as accretion of unregistered land to the riparian owner, and should not extend to registered land. The lower court and CA ruled in favour of defendants. ISSUES: 1. Is the change in the course of the Suague River sudden as claimed by the plaintiffs in order to negate the right of accretion by the defendants? 2. Is accretion under Art. 457 limited to unregistered lands, as such protecting registered lands against diminution in area? RULING: 1. NO. For a period of more than 40 years (before 1940 to 1980) the Suague River overflowed its banks yearly and the property of the defendant gradually received deposits of soil from the effects of the current of the river. The consequent increase in the area of Lot No. 7511 due to alluvion or accretion was possessed by the defendants whose tenants plowed and planted the same with corn and tobacco. Art. 457 provides that to the owners of lands 50 4S 2014-2015  

San Beda College of Law CIVIL LAW REVIEW   adjoining the banks of rivers belong the accretion which they gradually receive from the effects of the current of the waters. 2. NO. It also applies to registered lands. The rule that registration under the Torrens System does not protect the riparian owner against the diminution of the area of his registered land through gradual changes in the course of an adjoining stream is well settled. VDA. DE NAZARENO v. COURT OF APPEALS G.R. No. 98045, 26 June 1996 [PAGAYANAN] FACTS: The subject of this controversy is a parcel of land situated in Telegrapo, Puntod, Cagayan de Oro City. Said land was formed as a result of sawdust dumped into the dried-up Balacanas Creek and along the banks of the Cagayan River. Sometime in 1979, private respondents Jose Salasalan and Leo Rabaya leased the subject lots on which their houses stood from one Antonio Nazareno, petitioners’predecessor-in-interest. In the latter part of 1982, private respondents allegedly stopped paying rentals. As a result, Antonio Nazareno and petitioners filed a case for ejectment with the MTC of Cagayan de Oro. A decision was rendered against private respondents, which decision was affirmed by the RTC of Misamis Oriental. The case was remanded to the municipal trial court for execution of judgment after the same became final and executory. Private respondents filed a case for annulment of judgment before the RTC of Misamis Oriental, which dismissed the same. Subsequently, the decision of the lower court was finally enforced with the private respondents being ejected from portions of the subject lots they occu Before he died, Antonio Nazareno caused the approval by the Bureau of Lands of the survey plan with a view to perfecting his title over the accretion area being claimed by him. Before the approved survey plan could be released to the applicant, however, it was protested by private respondents before the Bureau of Lands. In compliance with the order of respondent District Land Officer Alberto M. Gillera, respondent Land Investigator Avelino G. Labis conducted an investigation and rendered a report to the Regional Director recommending that the survey plan in the name of Antonio Nazareno, be cancelled and that private respondents be directed to file appropriate public land applications. Based on said report, respondent Regional Director of the Bureau of Lands Roberto Hilario rendered a decision ordering the amendment of the survey plan in the name of Antonio Nazareno by segregating therefrom the areas occupied by the private respondents who, if qualified, may file public land applications covering their respective portions.

51 4S 2014-2015  

San Beda College of Law CIVIL LAW REVIEW   ISSUE: Is the subject private land, which was formed as a result of a sawdust dumped into the dried-up creek and along the banks of a river, can be considered as alluvion? RULING: NO. Court held that accretion, as a mode of acquiring property under Art. 457 of the Civil Code, requires the concurrence of these requisites: (1) that the deposition of soil or sediment be gradual and imperceptible; (2) that it be the result of the action of the waters of the river (or sea); and (3) that the land where accretion takes place is adjacent to the banks of rivers (or the sea coast). These are called the rules on alluvion which if present in a case, give to the owners of lands adjoining the banks of rivers or streams any accretion gradually received from the effects of the current of waters. For petitioners to insist on the application of these rules on alluvion to their case, the above-mentioned requisites must be present. However, they admit that the accretion was formed by the dumping of boulders, soil and other filling materials on portions of the Balacanas Creek and the Cagayan River bounding their land. It cannot be claimed, therefore, that the accumulation of such boulders, soil and other filling materials was gradual and imperceptible, resulting from the action of the waters or the current of the Balacanas Creek and the Cagayan River. The accretion was man-made or artificial. The requirement that the deposit should be due to the effect of the current of the river is indispensable. This excludes from Art. 457 of the Civil Code all deposits caused by human intervention. Putting it differently, alluvion must be the exclusive work of nature. In the case at bar, the subject land was the direct result of the dumping of sawdust by the Sun Valley Lumber Co. consequent to its sawmill operations. HEIRS OF EMILIANO NAVARRO v. INTERMEDIATE APPELLATE COURT G.R. No. 68166, 12 February 1997 [PARUNGAO] FACTS: Sinforoso Pascual filed an application to register and confirm his title to a parcel of land, situated in Sibocon, Balanga, Bataan. Pascual claimed that this land is an accretion to his property, situated in Barrio Puerto Rivas, Balanga, Bataan. It is bounded on the eastern side by the Talisay River, on the western side by the Bulacan River, and on the northern side by the Manila Bay. The Talisay River as well as the Bulacan River flow downstream and meet at the Manila Bay thereby depositing sand and silt on Pascual's property resulting in an accretion thereon. Sinforoso Pascual claimed the accretion as the riparian owner. Navarro thereupon filed an opposition to Pascual's application claiming that the land sought to be registered has always been part of the public domain, it being a part of the foreshore of Manila Bay; that he was a lessee and in 52 4S 2014-2015  

San Beda College of Law CIVIL LAW REVIEW   possession of a part of the subject property and that he had already converted the area covered by the lease into a fishpond. The case was decided adversely against Pascual. Thus, Pascual appealed to the Court of First Instance (now Regional Trial Court) of Balanga, Bataan. During the pendency of the trial of the consolidated cases, Emiliano Navarro and Pascual died and were substituted by their heirs. The court a quo rendered judgment finding the subject property to be foreshore land and, being a part of the public domain, it cannot be the subject of land registration proceedings. The heirs of Pascual appealed before the respondent appellate court which reversed the findings of the court a quo and granted the petition for registration of the subject property. ISSUE: Whether the disputed land is an accretion caused by the joint action of the Talisay and Bulacan Rivers which run their course on the eastern and western boundaries, respectively, of petitioners' own tract of land. RULING: Accretion as a mode of acquiring property under said Article 457, requires the concurrence of the following requisites: (1) that the accumulation of soil or sediment be gradual and imperceptible; (2) that it be the result of the action of the waters of the river; and (3) that the land where the accretion takes place is adjacent to the bank of the river. Accretion is the process whereby the soil is deposited, while alluvium is the soil deposited on the estate fronting the river bank; the owner of such estate is called the riparian owner. Riparian owners are, strictly speaking, distinct from littoral owners, the latter being owners of lands bordering the shore of the sea or lake or other tidal waters. The alluvium, by mandate of Article 457 of the Civil Code, is automatically owned by the riparian owner from the moment the soil deposit can be seen but is not automatically registered property, hence, subject to acquisition through prescription by third persons.

CO-OWNERSHIP: Rights of Each Co-owner   DEL BANCO v. INTERMEDIATE APPELLATE COURT G.R. No. 72694, 1 December 1987 [PULMA] FACTS: Three brothers, Benedicto Pansacola, Jose Pansacola and Manuel Pansacola (known as Fr. Manuel Pena) entered into an agreement which provided, among others: (1) That they will purchase from the Spanish Government the lands comprising the Island of Cagbalite which is located within the boundaries of the Municipality of Mauban, Province of Tayabas (now Quezon) and has an approximate area of 1,600 hectares; 53 4S 2014-2015  

San Beda College of Law CIVIL LAW REVIEW   (2) That the lands shall be considered after the purchase as their common property; (3) That the co-ownership includes Domingo Arce and Baldomera Angulo, minors at that time represented by their father, Manuel Pansacola (Fr. Manuel Pena) who will contribute for them in the proposed purchase of the Cagbalite Island; (4) That whatever benefits may be derived from the Island shall be shared equally by the co-owners in the following proportion: Benedicto Pansacola-1/4 share; Jose Pansacola-1/4 share; and, Domingo Arce and Baldomera Angulo2/4 shares which shall be placed under the care of their father, Manuel Pansacola (Fr. Manuel Pena). Co-owners entered into the actual possession and enjoyment of the Island purchased by them from the Spanish Government. They agreed to modify the terms and conditions of the agreement entered into by them on. The new agreement provided for a new sharing and distribution of the lands, comprising the Island of Cagbalite and whatever benefits may be derived therefrom. About one hundred years later, private respondents brought a special action for partition in the Court of First Instance of Quezon. Petitioners herein, interposed such defenses as prescription, res judicata, exclusive ownership, estoppel and laches. ISSUE: Can mere agreement to subdivide property terminate co-ownership? RULING: NO. It is not enough that the co-owners agree to subdivide the property. They must have a subdivision plan drawn in accordance with which they take actual and exclusive possession of their respective portions in the plan and titles issued to each of them accordingly (Caro v. Court of Appeals, 113 SCRA 10 [1982]). The mechanics of actual partition should follow the procedure laid down in Rule 69 of the Rules of Court. Maganon v. Montejo, 146 SCRA 282 [1986]). Neither can such actual possession and enjoyment of some portions of the Island by some of the petitioners herein be considered a repudiation of the coownership. It is undisputed that the Cagbalite Island was purchased by the original co-owners as a common property and it has not been proven that the Island had been partitioned among them or among their heirs. While there is co-ownership, a co-owner's possession of his share is co-possession which is linked to the possession of the other co-owners (Gatchalian v. Arlegui, 75 SCRA 234 [1977]). Furthermore, no prescription shall run in favor of a co-owner against his coowners or co-heirs so long as he expressly or impliedly recognizes the coownership (Valdez v. Olonga, 51 SCRA 71 [1973], Tero v. Tero, 131 SCRA 100 [1984]). Co-owners cannot acquire by prescription the share of the other co-owners, absent a clear repudiation of the co-ownership clearly 54 4S 2014-2015  

San Beda College of Law CIVIL LAW REVIEW   communicated to the other co-owners (Mariano v. De Vega, 148 SCRA 342 [1987]). An action for partition does not prescribe. Article 403 of the Old Civil Code, now Article 497, provides that the assignees of the co-owners may take part in the partition of the common property, and Article 400 of the Old Code, now Article 494 provides that each co-owner may demand at any time the partition of the common property, a provision which implies that the action to demand partition is imprescriptible or cannot be barred by laches (Budlong v. Pondoc, 79 SCRA 24 [1977]). An action for partition does not lie except when the coownership is properly repudiated by the co- owner (Jardin v. Hollasco, 117 SCRA 532 [1982]).

CO-OWNERSHIP: Obligations of Each Co-owner

  PARDELL v. BARTOLOME G.R. No. L-4656, 18 November 1912 [REMIGIO]

FACTS: Petitioner Vicenta Ortiz y Felin de Pardell and respondent Matilde Ortiz y Felin Bartolome were the existing heirs of the late Miguel Ortiz and Calixta Felin. On 1888, Matilde and co-defendant Gaspar de Bartolome y Escribano took it upon themselves without an judicial authorization or even extra judicial agreement the administration of the properties of the late Calixta and Miguel. These properties included a house in Escolta Street, Vigan, Ilocos Sur; a house in Washington Street, Vigan, Ilocos Sur; a lot in Magallanes Street, Vigan, Ilocos Sur; parcels of rice land in San Julian and Sta. Lucia; and parcels of land in Candon, Ilocos Sur. Vicenta filed an action in court asking that the judgement be rendered in restoring and returning to them one half of the total value of the fruits and rents, plus losses and damages from the aforementioned properties. However, respondent Matilde asserted that she never refused to give the plaintiff her share of the said properties. Vicenta also argued that Matilde and her husband, Gaspar are obliged to pay rent to the former for their occupation of the upper story of the house in Escolta Street. ISSUE: Is defendant Matilde Ortiz, as co-owner of the house on Calle Escolta, entitled, with her husband, to reside therein, without paying to her co-owner, Vicenta Ortiz, one-half of the rents which the upper story would have produced, had it been rented to a stranger? RULING: NO. The Court ruled that the spouses are not liable to pay rent. Their occupation of the said property was a mere exercise of their right to use the same as a co-owner. Each co-owner or tenant in common of undivided realty has the same rights therein as the others; he may use and enjoy the same 55 4S 2014-2015  

San Beda College of Law CIVIL LAW REVIEW   without other limitation except that he must not prejudice the rights of his coowners, but until a division is effected, the respective parts belonging to each can not be determined; each co-owner exercises joint dominion and is entitled to joint use. Matilde Ortiz and her husband occupied the upper story, designed for use as a dwelling, in the house of joint ownership; but the record shows no proof that, by so doing, Matilde occasioned any detriment to the interests of the community property, nor that she prevented her sister Vicenta from utilizing the said upper story according to her rights. It is to be noted that the stores of the lower floor were rented and an accounting of the rents was duly made to the plaintiffs.

CO-OWNERSHIP: Extinguishment of Co-ownership

  CARO v. COURT OF APPEALS G.R. No. L-48001, 25 March 1982 [RAMPAS]

FACTS: A parcel of land was co-owned by three siblings. One sold his undivided share to RP. Co-owners did not exercise their right of legal redemption, but widow of a deceased co-owner, as administratrix, moved for right of legal redemption. ISSUE: Can an administratrix of a deceased co-owner exercise right of legal redemption on the theory that the property has already been agreed to be divided among the co-owners? RULING: Once a property is partitioned, co-ownership is terminated and property ceases to be community. Inasmuch as the purpose of the law in establishing the right of legal redemption between co-owners is to reduce the number of participants until the community is done away with (Viola v. Tecson, 49 Phil. 808), once the property is subdivided and distributed among the co-owners, the community has terminated and there is no reason to sustain any right of legal redemption. BAILON-CASILAO v. COURT OF APPEALS No. L-78178, 15 April 1988 [SABILALA] FACTS: The petitioners herein filed a case for recovery of property and damages with notice of lis pendens on March 13, 1981 against the defendant and herein private respondent, Celestino Afable. The parcel of land involved in this case in the names of Rosalia, Gaudencio, Sabina, Bernabe, Nenita and Delia, all 56 4S 2014-2015  

San Beda College of Law CIVIL LAW REVIEW   surnamed Bailon, as co-owners, each with a 1/6 share. Gaudencio and Nenita are now dead, the latter being represented in this case by her children. Luz, Emma and Nilda. Bernabe went to China in 1931 and had not been heard from since then. It appears that on August 23, 1948, Rosalia Bailon and Gaudencio Bailon sold a portion of the said land consisting of 16,283 square meters to Donato Delgado. On May 13, 1949, Rosalia Bailon alone sold the remainder of the land consisting of 32,566 square meters to Ponciana V. Aresgado de Lanuza. On the same date, Lanuza acquired from Delgado the 16,283 square meters of land which the latter had earlier acquired from Rosalia and Gaudencio. On December 3, 1975, John Lanuza, acting under a special power of attorney given by his wife, Ponciana V. Aresgado de Lanuza, sold the two parcels of land to Celestino Afable, Sr. In his answer to the complaint filed by the herein petitioners, Afable claimed that he had acquired the land in question through prescription and contended that the petitioners were guilty of laches. ISSUE: Are petitioners guilty of laches as may effectively bar their present action? RULING: NO. Since a co-owner is entitled to sell his undivided share, a sale of the entire property by one co-owner without the consent of the other co-owners is not null and void. However, only the rights of the co-owner-seller are transferred, thereby making the buyer a co-owner of the property. The proper action in cases like this is not for the nullification of the sale or for the recovery of possession of the thing owned in common from the third person who substituted the co-owner or co-owners who alienated their shares, but the DIVISION of the common property as if it continued to remain in the possession of the co-owners who possessed and administered it. Neither recovery of possession nor restitution can be granted since the defendant buyers are legitimate proprietors and possessors in joint ownership of the common property claimed. In the light of the attendant circumstances, defendant-appellee’s defense of prescription is a vain proposition. Pursuant to Article 494 of the Civil Code, "(n)o co-owner shall be obliged to remain in the co-ownership. Such co-owner may demand at anytime the partition of the thing owned in common, insofar as his share is concerned.” In Budlong v. Bondoc [G.R. No. L-27702, September 9, 1977, 79 SCRA 24], this Court has interpreted said provision of law to mean that the action for partition is imprescriptible or cannot be barred by prescription. For Article 494 of the Civil Code explicitly declares: “No prescription shall lie in favor of a co-owner or co-heir so long as he expressly or impliedly recognizes the co-ownership.” As regards with Laches, it has been defined as the failure or neglect, for an unreasonable length of time to do that which by exercising due diligence could 57 4S 2014-2015  

San Beda College of Law CIVIL LAW REVIEW   or should have been done earlier; it is negligence or omission to assert a right within a reasonable time warranting a presumption that the party entitled to assert it either has abandoned it or declined to assert it. It must be noted that while there was delay in asserting petitioners’ rights, such delay was not attended with any knowledge of the sale nor with any opportunity to bring suit. In view of the lack of knowledge by the petitioners of the conduct of Rosalia in selling the land without their consent in 1975 and the absence of any opportunity to institute the proper action until 1981, laches may not be asserted against the petitioners. ROQUE v. INTERMEDIATE APPELLATE COURT G.R. No. 75886, 30 August 1988 [SANTILLAN] FACTS: The controversy here involves a parcel of land which was originally in the name of Januario Avendaño, a bachelor who died intestate and without issue. The intestate heirs of Avendafio executed a document through which an extrajudicial partition was effected among the intestate heirs. Some of the coowners then, all surnamed Avendaño, transferred their undivided threefourths (3/4) share to respondent Ernesto Roque and Victor Roque, thereby vesting in the latter full and complete ownership of the property. Subsequently, in an unnotarized deed Emesto and Victor Roque purportedly sold a three-fourths (3/4) undivided portion of the subject land to their half-sister, petitioner Concepcion Roque. The property, however, remained registered in the name of the decedent, Januario Avendaño. Upon the instance of petitioner and allegedly of respondent the land was surveyed. Consequent thereto, a Subdivision Plan was drawn. Petitioner claimed that preparation of the Subdivision Plan was a preliminary step leading eventually to partition. Respondents, however, refused to acknowledge petitioner's claim of ownership of any portion of Lot No. 1549 and rejected the plan to divide the land since the respondents alleged that they are the absolute owner of the subject land and that petitioner occupies a portion of the said property only by their mere tolerance. ISSUE: Does the right to seek an action for partition of the thing owned in common prescribes? RULING: NO. According to Article 494 of the Civil Code, "each co-owner may demand at any time the partition of the thing owned in common, insofar as his share is concemed." No matter how long the co-ownership has lasted, a co-owner can always opt out of the co-ownership, and provided the defendant co-owners or co-heirs have theretofore expressly or impliedly recognized the co-ownership, they cannot set up as a defense the prescription of the action for partition. But if the defendants show that they had previously asserted title in themselves adversely to the plaintiff and for the requisite period of time, the plaintiffs right to require recognition of his status as a co-owner will have been lost by prescription and the court cannot issue an order requiring partition. 58 4S 2014-2015  

San Beda College of Law CIVIL LAW REVIEW   In the case at bar, First of all, petitioner Concepcion Roque-the co-owner seeking partition — has been and is presently in open and continuous possession of a three-fourths (3/4) portion of the property owned in common. Second, prior to filing of the Civil Case, neither of the parties involved had asserted or manifested a claim of absolute and exclusive ownership over the whole of Lot No. 1549 adverse to that of any of the other co-owners: in other words, co-ownership of the property had continued to be recognized by all the owners. Consequently, the action for partition could not have and, as a matter of fact, had not yet prescribed at the time of institution by Concepcion of the action. DELIMA v. COURT OF APPEALS G.R. No. L-46296, 24 September 1991 [SAYO] FACTS: During his lifetime, Lino Delima acquired Lot No. 7758. Lino Delima later died leaving as his only heirs three brothers and a sister namely: Eulalio, Juanita, Galileo and Vicente, all surnamed Delima. After his death, TCT No. 2744 of the property in question was issued in the name of the Legal Heirs of Lino Delima, deceased represented by Galileo Delima. Galileo Delima, now substituted by respondents, executed an affidavit of “Extra-judicial declaration of Heirs.” Based on this affidavit, TCT No. 2744 was cancelled and TCT No. 3009 was issued on February 4, 1954 in the name of Galileo Delima alone to the exclusion of the other heirs. On February 29, 1968, petitioners filed an action for reconveyance and/or partition and for the annulment of TCT No. 3009 with damages. The RTC ruled in favor of petitioners, however on appeal, the CA reversed the decision of the RTC. Hence, this petition for review on certiorari. ISSUE: Is the action already barred by prescription, which shall entitle Galileo Delima to perfect his claim of ownership by acquisitive prescription to the exclusion of petitioners? RULING: YES. From the moment one of the co-owners claims that he is the absolute and exclusive owner of the properties and denies the others any share therein, the question involved is no longer one of partition but of ownership. In such a case, the imprescriptibility of the action for partition can no longer be invoked. It is settled that possession by a co-owner or co-heir is that of a trustee. In order that such possession is considered adverse to the cestui que trust amounting to a repudiation of the co-ownership, the following elements must concur: 1) that the trustee has performed unequivocal acts amounting to an ouster of the cestui que trust; 2) that such positive acts of repudiation had 59 4S 2014-2015  

San Beda College of Law CIVIL LAW REVIEW   been made known to the cestui que trust; and 3) that the evidence thereon should be clear and conclusive. Since an action for reconveyance of land based on implied or constructive trust prescribes after ten (10) years, it is from the date of the issuance of such title that the effective assertion of adverse title for purposes of the statute of limitations is counted, or in this case from February 4, 1954. As the certificate of title was notice to the whole world of his exclusive title to the land, such rejection was binding on the heirs and started as against them the period of prescription. Hence, when petitioners filed their action for reconveyance on February 29, 1968, such action was already barred by prescription. Whatever claims the other co-heirs could have validly asserted before can no longer be invoked by them at this time. AGUILAR v. COURT OF APPEALS G.R. No. 76351, 29 October 1993 [SENTILLAS] FACTS: Virgilio and Senen are brothers; Virgilio is the youngest of seven (7) children of the late Maximiano Aguilar, while Senen is the fifth. On 28 October 1969, the two brothers purchased a house and lot in Parañaque where their father could spend and enjoy his remaining years in a peaceful neighborhood. Initially, the brothers agreed that Virgilio's share in the co-ownership was twothirds while that of Senen was one-third. By virtue of a written memorandum, Virgilio and Senen agreed that henceforth their interests in the house and lot should be equal, with Senen assuming the remaining mortgage obligation of the original owners with the Social Security System (SSS) in exchange for his possession and enjoyment of the house together with their father. Since Virgilio was then disqualified from obtaining a loan from SSS, the brothers agreed that the deed of sale would be executed and the title registered in the meantime in the name of Senen. It was further agreed that Senen would take care of their father and his needs since Virgilio and his family were staying in Cebu. After Maximiano Aguilar died in 1974, Virgilio demanded from Senen that the latter vacate the house and that the property be sold and proceeds thereof divided among them. Senen refused, prompting Virgilio to file an action to compel the sale of the house and lot so that the they could divide the proceeds between them. The trial court found Virgilio and Senen to be co-owners of the house and lot, in equal shares on the basis of their written agreement. However, it ruled that Virgilio has been deprived of his participation in the property by Senen's continued enjoyment of the house and lot, free of rent, despite demands for rentals and continued maneuvers to delay partition. The trial court also upheld the right of Virgilio as co-owner to demand partition. Since Virgilio could not 60 4S 2014-2015  

San Beda College of Law CIVIL LAW REVIEW   agree to the amount offered by Senen for the former's share, the trial court held that this property should be sold to a third person and the proceeds divided equally between the parties. The trial court likewise ordered defendant to vacate the property and pay plaintiff P1,200.00 as rentals from January 1975 up to the date of decision plus interest from the time the action was filed. ISSUES: 1.May a co-owner demand the sale of a house and lot held in common with other co-owners? 2. Should a co-owner who enjoys possession of the thing held in common pay rentals to other co-owners? RULING: 1. YES. Article 494 of the Civil Code provides that no co-owner shall be obliged to remain in the co-ownership, and that each co-owner may demand at any time partition of the thing owned in common insofar as his share is concerned. Corollary to this rule, Art. 498 of the Code states that whenever the thing is essentially, indivisible and the co-owners cannot agree that it be, allotted to one of them who shall indemnify the others, it shall be sold and its proceeds accordingly distributed. This is resorted to a. when the right to partition the property is invoked by any of the co-owners but because of the nature of the property it cannot be subdivided or its subdivision would prejudice the interests of the co-owners, and b. the co-owners are not in agreement as to who among them shall be allotted or assigned the entire property upon proper reimbursement of the co-owners. 2. NO. Each co-owner of property held pro indiviso exercises his rights over the whole property and may use and enjoy the same with no other limitation than that he shall not injure the interests of his co-owners, the reason being that until a division is made, the respective share of each cannot be determined and every co-owner exercises, together with his co-participants joint ownership over the pro indiviso property, in addition to his use and enjoyment of the same. ADDITIONAL FACTS PECULIAR TO THE CASE: Although the Court did rule that “each co-owner of property... may use and enjoy the same with no other limitation than that he shall not injure the interests of his co-owners,” the respondent was still made to pay monthly rentals because he failed to vacate the house and lot despite order of the trial court. He was ordered to pay rentals from the time the trial court ordered him to vacate for the use and enjoyment of the other half of the property appertaining to petitioner.

61 4S 2014-2015  

San Beda College of Law CIVIL LAW REVIEW   TOMAS CLAUDIO MEMORIAL COLLEGE v. COURT OF APPEALS G.R. No. 124262, 12 October 1999 [SINOCRUZ] FACTS: Private respondents De Castros filed an action for Partition before the RTC alleging that their predecessor-in-interest, Juan, died intestate in 1993 and they are his only surviving legitimate heirs. They also alleged that their father owned a parcel of land designated as Lot No. 3010 in Morong, Rizal. They further claimed that without their knowledge and consent, said lot was sold by their brother Mariano to petitioner Tomas Claudio Memorial College (TCMC) when the former represented himself to be the sole heir. The De Castros contend that the sale should only affect Mariano’s undivided share but not the shares of the other co-owners. TCMC filed a motion to dismiss contending lack of jurisdiction and prescription and/or laches. ISSUE: Are petitioners De Castros’ right to partition barred by prescription? RULING: NO. Article 493 of the Civil Code provides that the sale or other disposition affects only the seller’s share pro indiviso, and the transferee gets only what correspondents to his grantor’s share in the partition owned in common. Since a co-owner is entitled to sell his undivided share, a sale of the entire property by one co-owner without the consent of the other coowners is not null and void. However, only the rights of the co-owner/seller are transferred, making the buyer a co-owner. The proper action in a case like this is the division or partition of the entire property that continued to remain in the possession of the co-owners. With respect to partition, Article 494 of the Civil Code provides that “no coowner shall be obliged to remain in the co-ownership. Such co-owner may demand at any time the partition of the thing owned in common, insofar as his share is concerned. xxx. No prescription shall lie in favor of a co-owner or coheirs as long as he expressly or impliedly recognizes that co-ownership.” ROBLES v. COURT OF APPEALS G.R. Number 123509, 14 March 2000 [TILOS] FACTS: The property subject of this case is originally owned by Leon Robles. When he died, it passed to his son Silvino who declared the property in his name and paid the taxes thereon. Upon the latter’s death, his widow and children inherited the property. Petitioners Lucio Robles, et al. were the children of Silvino, and Hilario Robles is their half-brother. The task of cultivating was assigned to Lucio while the payment of the land taxes was entrusted to Hilario. For unknown reason, the tax declaration of the parcel of land in the 62 4S 2014-2015  

San Beda College of Law CIVIL LAW REVIEW   name of Silvino was cancelled and transferred to Exequiel Ballena. Ballena secured a loan from Antipolo Rural Bank using the tax declaration as security. Somehow the tax declaration was transferred to the name of Antipolo Rural Bank and later was transferred to the name of respondentspouses Hilario and Andrea Robles. Andrea secured a loan from Cardona Rural Bank using the tax declaration as security. For failure to pay the mortgage debt, the property was foreclosed with Cardona Rural Bank emerging as the highest bidder. The bank sold the property to spouses Vergel and Ruth Santos. In Sept. 1987, petitioners discovered the mortgage and attempted to redeem the property but was unsuccessful. In 1988, the spouses Santos took possession of the propertry and was able to secure a Free Patent. Petitioners then filed an action for quieting of title. Respondents questioned their standing to sue for quieting of title, contending that petitioners no longer have any interest to the property in question due to the mortgage effected by Hilario and the consequent foreclosure thereof by the Bank. Respondents argued that Hilario had become the absolute owner of the property at the time he mortgaged the same. The CA ruled that the several transfers of the tax declaration of the property in question from Silvino until to the spouses Santos had the effect of divesting petitioners of their title by prescription to Hilario. ISSUES: 1. Do the petitioners have appropriate title that will entitle them to the remedy of the quieting of title? 2. Did Hilario acquire the share of his co-owners in the disputed property by prescription? RULING: 1. YES. An action to quiet title, under Art. 476, NCC, is a common-law remedy for the removal of any cloud or doubt or uncertainty on the title to real property. It is essential for the plaintiff or complainant to have a legal or an equitable title to or interest in the real property which is the subject matter of the action. Also, the deed, claim, encumbrance or proceeding that is being alleged as a cloud on plaintiff's title must be shown to be in fact invalid or inoperative despite its prima facie appearance of validity or legal efficacy. That there is an instrument or a document which, on its face, is valid and efficacious is clear in the present case. Petitioners allege that their title as owners and possessors of the disputed property is clouded by the tax declaration and, subsequently, the free patent thereto granted to Spouses Santos. Petitioners anchor their claim on their open and continuous possession as owners. Spouses Santos, on the other hand, trace their claims to Exequiel, and then to Hilario who mortgaged the same to the Bank as absolute owner. It was from Exequiel that Hilario’s claim is rooted. However, in this case, there is a failure to show Exequiel’s title to the property in question. When Hilario, therefore, mortgaged the property, he did so in his capacity as mere co-owner thereof. Consequently, the said transaction did not divest the petitioner of the title to the property at the time of the institution of the complaint for quieting of title. 63 4S 2014-2015  

San Beda College of Law CIVIL LAW REVIEW   2. NO. Hilario effected no clear and evident repudiation of the co-ownership. It is a fundamental principle that a co-owner cannot acquire by prescription the share of the other co-owners, absent any clear repudiation of the coownership. In order that the title may prescribe in favor of a co-owner, the following requisites must concur: (1) the co-owner has performed unequivocal acts of repudiation amounting to an ouster of the other co-owners; (2) such positive acts of repudiation have been made known to the other co-owners; and (3) the evidence thereof is clear and convincing. In the present case, Hilario did not have possession of the subject property; neither did he exclude the petitioners from the use and the enjoyment thereof, as they had indisputably shared in its fruits. Likewise, his act of entering into a mortgage contract with the bank cannot be construed to be a repudiation of the coownership. As absolute owner of his undivided interest in the land, he had the right to alienate his share, as he in fact did. Neither should his payment of land taxes in his name, as agreed upon by the co-owners, be construed as a repudiation of the co-ownership. The assertion that the declaration of ownership was tantamount to repudiation was belied by the continued occupation and possession of the disputed property by the petitioners as owners.

POSSESSION: Classifications of Possession

  RIZAL CEMENT CO., INC., v. CONSUELO C. VILLAREAL, ISABEL C. VILLAREAL, FLAVIANO C. VILLAREAL, ALFREDO V. GOMEZ, AURORA V. GOMEZ AND THE COURT OF APPEALS Gr. No. L-30272, 28 February 1985 [ABIERA]

FACTS: Private respondents filed with the then Court of First Instance of Rizal in Pasig, an Application for Registration, alleging that they are the owners of two agricultural lots bounded and described as shown on plan Psd-147662 as Lots Nos. 1 and 2. Based on respondents' testimonial and documentary evidence, it appears that the subject lots were originally belonged to one Maria Certeza; that upon her death, the property was involved in a litigation between her grandchildren and Gonzalo Certeza and that the lots were given by the latter to former Justice de Joya as the latter's attorney's fees; that the lots were then sold by de Joya to Filomeno Sta. Ana who, in turn sold the same to spouses Victoriano Cervo and Ignacia Guillermo; and that sometime in November 1955, the said spouses sold the said lots to the herein respondents as shown by a duly notarized deed of sale. On the other hand, petitioner Rizal Cement Company filed an opposition, claiming to be the owner of the subject lots, having bought the same from Maria Certeza, and to have been in continuous and adverse possession of the property since 1911. To substantiate its claim, petitioner submitted documentary evidence, the most important of which are the following — 1) Plan Psu-2260 covering the subject lots; 2) A sketch plan of the geographical 64 4S 2014-2015  

San Beda College of Law CIVIL LAW REVIEW   position of the real pro-parties of Madrigal and Company; 3) Tax declaration; and 4) Real estate receipts. During trial, witnesses for the respondents testified that petitioner did not take possession of the land and that it was Maria Certeza who had the possession of the land until her death and that the tenants gave the harvest of the land to Maria Certeza. After trial, the CFI denied the application for registration of respondents and ordered the issuance of a decree of registration in the name of Rizal Cement Co. after finality of said decision. Respondents appealed to the Court of Appeals, which reversed and set aside the CFI’s decision in favor of the respondents. The Court of Appeals denied Rizal’s Motion for Reconsideration, hence this petition. ISSUE: Did the respondents possess the property in the concept of an owner? RULING: YES. Being an attribute of ownership, appellants' possession of the land in question goes far to tip the scale in their favor. The right to possess flows from ownership. No person will suffer adverse possession by another of what belongs to him. Were the petitioner rightful owner of the land in question, it would not have allowed the tenants to cultivate the land and give the owner's share to appellants and/or their predecessors. It would have opposed the survey for respondents' vendors but did not as shown in the surveyor's certificate. If petitioner really bought Lot 2 from Maria Certeza as claimed, it has not been explained how she could sell a portion thereof to Apolonia Francisco, married to Valentin Marquez, as confirmed by the husband in his deposition who as employee of oppositor would have known of its acquisition. As correctly held by the Court of Appeals, respondents possess the property in the concept of an owner. Possession is acquired by the material occupation of a thing or the exercise of a right or by the fact it is subject to the action of our will, or by the proper acts and legal formalities established for acquiring such right. Petitioner's evidence, consisting of tax receipts, tax declaration and survey plan are not conclusive and indisputable basis of one's ownership of the property in question. Assessment alone is of little value as proof of title. Mere tax declaration does not vest ownership of the property upon the declarant. Settled is the rule that neither tax receipts nor declaration of ownership for taxation purposes alone constitutes sufficient evidence of ownership or of the right to possess realty. They must be supported by other effective proofs. Neither can the survey plan or technical descriptions prepared at the instance of the party concerned be considered in his favor, the same being self-serving.

65 4S 2014-2015  

San Beda College of Law CIVIL LAW REVIEW   IGNACIO WONG v. HON. LUCAS D. CARPIO G.R. No. L-50264, 21 October 1991 [ALDUESO]

FACTS: The CFI of Davao Del Sur presided by the public respondent reversed the ruling of the Municipal Court of Sta. Maria, Davao del Sur in an action for forcible entry (FE) ordering the dismissal of the complaint as well as the counterclaim. Manuel Mercado (herein private respondent, plaintiff in lower court) acquired his rights to possess the land in litigation which is particularly described and embraced in Transfer Certificate of title No. (T-4244) T-972 from William Giger by virtue of a deed of sale with right to repurchase which was executed in 1972 for a consideration of P3,500.00. Then, in 1973, William Giger again asked an additional amount of P2,500.00 from plaintiff and so he required William Giger to sign a new deed of Pacto de Retro Sale. In 1972, plaintiff began harvesting only the coconut fruits and he paid the taxes on the landfor Mr. Giger. He went periodically to the land to make copra but he never placed any person on the land in litigation to watch it. Neither did he reside on the land as he is a businessman and storekeeper by occupation and resides at Lower Sta. Maria, Davao del Sur while the land in litigation is at Colongan, Sta. Maria. Neither did he put any sign or hut to show that he is in actual possession. He knew defendants' laborers were in the land in suit as early as August, 1976 and that they have a hut there but he did not do anything to stop them. Instead plaintiff was happy that there were people and a hut on the land in suit. Therein defendant Ignacio Wong went to the land in litigation to find out if there were other people residing there or claiming it besides the owner and he found none. So, in July, 1976, defendant Ignacio Wong bought the parcel of land in litigation from William Giger and his wife Cecilia Valenzuela (Exhibit 5). After the execution of Exhibit 5, defendant Ignacio Wong asked for the delivery of the title to him and so he has in his possession TCT No. (T-4244) T-974 in the name of William Giger. Mr. Wong declared the land in suit for taxation purposes in his name. He tried to register the pacto de retro sale with the Register of Deeds by paying the registration fee but due to some technicalities, the pacto de retro sale could not be registered. The defendant Wong placed laborers on the land in suit, built a small farm house after making some clearings and fenced the boundaries. He also placed signboards. On September 27, 1976, plaintiff Manuel Mercado again went to the land in suit to make copras. That was the time the matter was brought to the attention of the police of Sta. Maria, Davao del Sur and the incident entered in the police blotter. Wong ordered the hooking of the coconuts from the land in litigation and nobody disturbed him. But on November 29, 1976, defendant (Wong) received a copy of plaintiff's complaint for forcible entry with summons to answer which is the case now before the Court.

66 4S 2014-2015  

San Beda College of Law CIVIL LAW REVIEW   The CFI finds the plaintiff (Mercado) to have taken possession of the property earlier in point of time and defendant (Wong) is an intruder and must, as he is hereby ordered to return, the possession of the land in question for the plaintiff, paying a monthly rental of P400.00 from August, 1976, till the property is returned with costs against the defendant. Judgment is reversed. Herein petitioner contends that respondent has not established prior possession and that private respondent's periodic visit to the lot to gather coconuts may have been consented to and allowed or tolerated by the owner thereof for the purposes of paying an obligation that may be due to the person gathering said nuts and that a person who enters a property to gather coconut fruits and convert the same to copras may only be a hired laborer who enters the premises every harvest season to comply with the contract of labor with the true owner of the property. ISSUES: 1. Is the petitioner a possessor in good faith so as to allow him avoid paying rentals to private respondent? 2. Is the petitioner correct in stating that respondent did not have prior possession of the property? RULING: 1. NO. A perusal of the records of the case shows that petitioner received private respondent's complaint for forcible entry with summons on November 29, 1976. His good faith therefore ceased on November 29,1976. Accordingly, the computation of the payment of monthly rental should start from December, 1976, instead of August, 1976. It should be noted that possession acquired in good faith does not lose this character except in the case and from the moment facts exist which show that the possessor is not unaware that he possesses the thing improperly or wrongfully. (Art. 528, Civil Code). Possession in good faith ceases from the moment defects in the title are made known to the possessors, by extraneous evidence or by suit for recovery of the property by the true owner. Whatever may be the cause or the fact from which it can be deduced that the possessor has knowledge of the defects of his title or mode of acquisition, it must be considered sufficient to show bad faith. 2. YES. It should be stressed that "possession is acquired by the material occupation of a thing or the exercise of a right, or by the fact that it is subject to the action of our will, or by the proper acts and legal formalities for acquiring such right." (Art. 531, Civil Code; Rizal Cement Co., Inc. v. Villareal, 135 SCRA 15 [1985]); and that the execution of a sale thru a public instrument shall be equivalent to the delivery of the thing, unless there is stipulation to the contrary . . . . If, however, notwithstanding the execution of the instrument, the purchaser cannot have the enjoyment and material tenancy of the thing and make use of it herself, because such tenancy and enjoyment are opposed by another, then delivery has not been effected. (Paras, Civil Code of the Philippines, Vol. II, 1989 Ed., p. 400). 67 4S 2014-2015  

San Beda College of Law CIVIL LAW REVIEW   Applying the above pronouncements on the instant case, it is clear that possession passed from vendor William Giger to private respondent Manuel Mercado by virtue of the first sale a retro, and accordingly, the later sale a retro in favor of petitioner failed to pass the possession of the property because there is an impediment — the possession exercised by private respondent. Possession as a fact cannot be recognized at the same time in two different personalities except in the cases of co-possession. Should a question arise regarding the fact of possession, the present possessor shall be preferred; if there are two possessions, the one longer in possession, if the dates of possession are the same, the one who presents a title; and if these conditions are equal, the thing shall be placed in judicial deposit pending determination of its possession or ownership through proper proceedings (Art. 538, Civil Code). SOMODIO v. COURT OF APPEALS G.R. No. 82680, 15 August 1994 [ALON] FACTS: Somodio, petitioner herein, owned a lot numbered 6328-X, over which he planted ipil-ipil trees, coconut trees and other fruit-bearing trees. In 1976, he started the construction of a building on the property. The construction was, however, unfinished because he had to leave for Kidapawan for his employment and visited the property only intermittently. He allowed respondent Ayco, to transfer his hut to petitioner's lot. About six years later, petitioner demanded that Ayco vacate the premises but such demand proved futile. Hence, Samodio filed an action for unlawful detainer with damages against respondent Ayco. Meanwhile, the other respondent Purisima entered the land and constructed a house thereon claiming that Lot No. 6328-X was in payment of the fee for the services of his father, for having father surveyed the land and that he caused the construction of a perimeter wall in the area. Somodio, thereafter, filed against respondent Purisima a complaint for forcible entry before the same court. The MTC rendered a decision finding that respondent Purisima built his house "almost on the spot where Somodio's unfinished house" stood "thru stealth and strategy," not knowing that the house was built on Lot No. 6328-X and not on Lot No. 6328-Y Subsequently, the court did not believe respondent Ayco's claim that the administratrix of the estate of respondent Purisima's father authorized him to build a hut on Lot No. 6328-X in 1976. RTC affirmed in toto the decision of the MTC. The CA, however, held that herein petitioner had not "clearly and conclusively established physical, prior possession over Lot No. 6328-X." ISSUE: Did Somodio establish clearly and conclusively his physical and prior possession over Lot No. 6328-X? RULING: YES. Possession in the eyes of the law does not mean that a man has to have his feet on every square meter of ground before it can be said that he is 68 4S 2014-2015  

San Beda College of Law CIVIL LAW REVIEW   in possession. It is sufficient that petitioner was able to subject the property to the action of his will. Article 531 of the Civil Code of the Philippines provides: Possession is acquired by the material occupation of a thing or the exercise of a right, or by the fact that it is subject to the action of our will, or by the proper acts and legal formalities established for acquiring such right.

Even if the Court of Appeals is correct in its finding that petitioner started introducing improvements on the land only in 1981, he still enjoyed priority of possession because respondent Purisima entered the premises only in 1983. Neither did he present proof that between 1958, when his father allegedly took possession of the land, and 1983, when said respondent himself entered the land, his father ever exercised whatever right of possession he should have over the property. Under these circumstances, priority in time should be the pivotal cog in resolving the Issue of possession. Petitioner's prior possession over the property, however, is not synonymous with his right of ownership over the same. As earlier stated, resolution of the issue of possession is far from the resolution of the issue of ownership. Forcible entry is merely a quieting process and never determines the actual title to an estate. GAVINA MAGLUCOT-AW, CATALINA ORCULLO, RICHARD ESTANO, NIDA MAGLUCOT, MELANIA MAGLUCOT-CATUBIG, EMILIANO CATUBIG, LADISLAO SALMA v. LEOPOLDO MAGLUCOT, SEVERO MAGLUCOT, WILFREDA MAGLUCOT-ALEJO AND CONSTANCIO ALEJO G.R. No. 132518, 28 March 2000 [BELARMINO] FACTS: Petitioners filed with the RTC a complaint for recovery of possession and damages alleging that they are the owners of Lot No. 1639-D. Said lot was originally part of Lot No. 1639 which was covered by OCT No. 6775 issued in the names of Hermogenes Olis, Bartolome Maglucot, Pascual Olis, Roberto Maglucot, Anselmo Lara and Tomas Maglucot. Tomas Maglucot, one of the registered owners and respondents’ predecessor-in-interest, filed a petition to subdivide the said Lot. Consequently, the CFI of Negros Oriental issued an order directing the parties to subdivide the lot into 6 portions. Sometime in 1963, Guillermo Maglucot rented a portion of Lot No. 1639-D. Subsequently, Leopoldo and Severo, both surnamed Maglucot, rented portions of the same lot in 1964 and 1969, respectively. In December 1992, however, the said respondents stopped paying rentals claiming ownership over the subject lot. Petitioners thus filed a complaint for the recovery of possession of Lot No. 1639-D and damages. Petitioners assert that respondents are estopped from claiming to be coowners of the subject lot in view of the mutual agreement in 1946, judicial 69 4S 2014-2015  

San Beda College of Law CIVIL LAW REVIEW   confirmation in 1952, and respondents’ acquiescence because they themselves exclusively exercised ownership over Lot No. 1639-A beginning 1952 up to the present. For their part, respondents posit three points in support of their position. First, they emphasize that petitioners failed to show that the interested parties were apprised or notified of the tentative subdivision contained in the sketch and that the CFI subsequently confirmed the same. Second, they point to the fact that petitioners were unable to show any court approval of any partition. Third, they maintain that Lot No. 1639 remain undivided since to date, OCT No. 6275 is still an existing and perfectly valid title, containing no annotation of any encumbrance or partition whatsoever. The lower court rendered judgment in favor of petitioners. The RTC found the existence of tax declarations in the names of Hermogenes Olis and Pascual Olis (purported owners of Lot Nos. 1639-A and 1639-B, respectively) as indubitable proof that there was a subdivision of Lot No. 1639. It likewise found that Tomas Maglucot, respondents’ predecessor-in-interest, took active part in the partition as it was he, in fact, who commenced the action for partition.The court a quo cited Article 1431 of the Civil Code which states that "[t]hrough estoppel an admission or representation is rendered conclusive upon the person making it, and cannot be denied or disproved as against the person relying thereon." ISSUE: Whether parties to a partition proceeding, who elected to take under partition, and who took possession of the portion allotted to them, are estopped from questioning the title to partition alloted to another party RULING: YES. Parties to a partition proceeding, who elected to take under partition, and who took possession of the portion allotted to them, are estopped to question title to portion allotted to another party. A person cannot claim both under and against the same instrument. In other words, they accepted the lands awarded them by its provisions, and they cannot accept the decree in part, and repudiate it in part. They must accept all or none. Parties who had received the property assigned to them are precluded from subsequently attacking its validity of any part of it. Here, respondents, by themselves and/or through their predecessors-in-interest, already occupied of the lots in accordance with the sketch plan. This occupation continued until this action was filed. They cannot now be heard to question the possession and ownership of the other co-owners who took exclusive possession of Lot 1639D also in accordance with the sketch plan. CEQUENA V. BOLANTE G.R. No. 137944, 6 April 2000 FACTS: The land subject of the case was formerly declared for taxation purposes in the name of Sinforoso Mendoza prior to 1954 but is now declared in the name 70 4S 2014-2015  

San Beda College of Law CIVIL LAW REVIEW   of Margarito Mendoza. Petitioners are the daughters of Margarito Mendoza while the respondent is the only daughter of Sinforoso Mendoza. Margarito Mendoza and Sinforoso Mendoza were brothers, now deceased. During the cadastral survey of the property on October 15, 1979 there was already a dispute between Honorata M. Bolante and Miguel Mendoza, brother of petitioners. Respondent was occupying the property in question. ISSUE: Who is the lawful owner and possessor of the land subject of the case RULING: The respondents. Despite their dispossession in 1985, the petitioners did not lose legal possession because possession cannot be acquired through force or violence. To all intents and purposes, a possessor, even if physically ousted, is still deemed the legal possessor. Indeed, anyone who can prove prior possession, regardless of its character, may recover such possession. Respondent's possession was not disturbed until 1953 when the petitioners' father claimed the land. But by then, her possession, which was in the concept of owner -- public, peaceful, and uninterrupted-- had already ripened into ownership. Furthermore she herself, after her father's demise, declared and paid realty taxes for the disputed land. Tax receipts and declarations of ownership for taxation, when coupled with proof of actual possession of the property, can be the basis of a claim for ownership through prescription. In contrast, the petitioners, despite thirty-two years of farming the subject land, did not acquire ownership. It is settled that ownership cannot be acquired by mere occupation. Unless coupled with the element of hostility toward the true owner, occupation and use, however long, will not confer title by prescription or adverse possession. Moreover, the petitioners cannot claim that their possession was public, peaceful and uninterrupted. Although their father and brother arguably acquired ownership through extraordinary prescription because of their adverse possession for thirty-two years (1953-1985), this supposed ownership cannot extend to the entire disputed lot, but must be limited to the portion that they actually farmed.

POSSESSION: Loss of Possession

  ARAGON v. THE INSULAR GOVERNMENT G.R. No. L-6019, 25 March 1911 [BONAVENTE]

FACTS: In 1892 a possessory title to the land in question was duly registered in favor of Inocencio Aragon, one of the predecessors in interest of the applicants Juan Aragon, et.al. For a long period of years, the applicants and their predecessors in interest have been in possession of the parcel of land in question, under and undisputed claim of ownership. It is located toward the 71 4S 2014-2015  

San Beda College of Law CIVIL LAW REVIEW   center of one of the most valuable residential sections of the city of Manila, and that for many years a house stood upon this land, and was occupied by some of the predecessors in interest of the applicants in these proceedings. There are strong reasons to believe that the land in question was originally well above the ebb and flow of the tide and that only in later years have the waters risen to such a height along the shores of the Bay of Manila at this point as to cover the land in question completely at high tide, though it does not definitely appear whether this is due to changes in the current and flow of the waters in the bay, or to the gradual sinking of the land along the coast. The Court of Land Registration adjudicated title over the land in favor of the applicants and ordering its registry in accordance with the provisions of "The Land Registration Act." The Government of the Philippine Islands , through its proper representatives, objected to the application for registry on the ground that, as it alleges, the land in question is a part of the public domain. As alleged by the opponent, the land in question, at the time when the trial was had in the court below, was so located that at high tide it was completely covered by the waters of the Bay of Manila, though the receding waters left it completely bare at low tide. ISSUE: Did the owners lose their possession of the land in question as provided under Article 555 of the New Civil Code? RULING: NO. If the Government is justified in disturbing the possession of the applicants, it can only be on the ground that they have abandoned their property, or that it has been totally destroyed and has now become a part of the public domain by the erosive action of the sea. It is quite clear that applicants have never abandoned their possession under a claim of ownership of this land. There has been no such destructive or total loss of the property as would justify a holding that the owners have lost possession. Doubtless the property has been injured by the erosive action of the sea. Doubtless the owners in order to profitably enjoy the possession of this property will be compelled to make some relatively small expenditures by way of a "fill" or a retaining wall. If the applicants have not lost their right of possession, the Government's claim of ownership, on the ground that this is a part of the playa (shore) of Manila Bay, necessarily falls to the ground. It affirmatively appears that the owners of the land in question have never in fact nor in intent abandoned it, and that keeping in mind its location and actual condition it can not be said to have been totally destroyed for the purposes for which it was held by them, so as to have become a part of the playa (shore) of the Bay of Manila.

72 4S 2014-2015  

San Beda College of Law CIVIL LAW REVIEW   CATHOLIC VICAR APOSTOLIC OF THE MOUNTAIN PROVINCE v. COURT OF APPEALS G.R. NO. 80294-95, 21 September 1988 [CABBUAG] FACTS: The whole controversy started when the herein petitioner filed an application for registration of lands 1, 2, 3 and 4 in La Trinidad, Benguet on September 5, 1962. The heirs of Juan Valdez and the heirs of Egmidio Octaviano filed an opposition on lots 2 and 3, respectively. On November 17, 1965, the land registration court confirmed the registrable title of the petitioner. On May 9, 1977, the Court of Appeals reversed the decision and dismissed the Vicar’s application. The heirs filed a motion for reconsideration, praying that the lots be ordered registered under their names. The Court of Appeals denied the motion for lack of sufficient merit. Both parties then came before the Supreme Court. The Supreme Court, in a minute resolution, denied both petitions. The heirs filed the instant cases for the recovery and possession of the lots. Respondents argue that the petitioner is barred from setting up the defense of ownership or long and continuous possession by the prior judgment of the Court of Appeals under the principle of res judicata. Petitioner contends that the principle is not applicable because the dispositive portion of the judgment merely dismissed the application for registration. ISSUE: Did ownership of Lots 2 and 3 transfer to petitioner Vicar through acquisitive prescription? RULING: NO. Private respondents were able to prove that their predecessors' house was borrowed by petitioner Vicar after the church and the convent were destroyed. They never asked for the return of the house, but when they allowed its free use, they became bailors in commodatum and the petitioner the bailee. The bailees' failure to return the subject matter of commodatum to the bailor did not mean adverse possession on the part of the borrower. The bailee held in trust the property subject matter of commodatum. The adverse claim of petitioner came only in 1951 when it declared the lots for taxation purposes. The action of petitioner Vicar by such adverse claim could not ripen into title by way of ordinary acquisitive prescription because of the absence of just title.

73 4S 2014-2015  

San Beda College of Law CIVIL LAW REVIEW  

POSSESSION: Theory Exceptions Thereto

of

Irrevindicability

and

  EDCA PUBLISHING & DISTRIBUTING CORP v. SANTOS G.R. No. 80298, 26 April 1990 [CRON] FACTS: Jose Cruz ordered 406 books from EDCA through telephone, payable on delivery. Cruz issued a personal check covering the purchase price for the books. Cruz sold 120 books to Santos who, upon verifying the seller’s ownership from the invoice he showed her, paid him the purchase price. Upon investigation, EDCA confirmed that Cruz was an impostor and had no intention of paying the books ordered (the check issued bounced). Thus, with the assistance of police, EDCA forcibly seized the books from Santos and threatened her with prosecution for buying stolen property. Santos sued for the recovery of the books after demand for their return was rejected by EDCA. EDCA argued that pursuant to Article 559 of the Civil Code, it has the right to recover the books since it was unlawfully deprived thereof. ISSUE: Was EDCA unlawfully deprived of the books because the check issued by the impostor in payment therefor was dishonored? RULING: NO. There was no unlawful deprivation of property which would entitle the petitioner to recover a property from the person possessing it in good faith. Actual delivery of the books having been made, Cruz acquired ownership over the books which he could then validly transfer to the private respondents. The fact that he had not yet paid for them to EDCA was a matter between him and EDCA and did not impair the title acquired by the private respondents to the books. Non-payment only creates a right to demand payment or to rescind the contract, or to criminal prosecution in the case of bouncing checks. But absent the stipulation above noted, delivery of the thing sold will effectively transfer ownership to the buyer who can in turn transfer it to another. DE GARCIA v. COURT OF APPEALS G.R. No. L-20264, 30 January 1971 [CAINDAY] FACTS: Guevarra was the owner of a lady’s diamond ring. It was stolen from her house. Later on, while she was talking to Garcia, an owner of a restaurant, 74 4S 2014-2015  

San Beda College of Law CIVIL LAW REVIEW   she recognized the ring worn by Garcia as her lost ring. Garcia allege to have bought it from her comadre. After ascertaining that the ring was hers, Guevarra demanded the same from Garcia but Garcia refused to return the same. ISSUE: What can be a valid defense of a possessor of a thing which is alleged to have been stolen or lost, against the owner who wishes to recover the same? RULING: One who has lost or has been unlawfully deprived of a movable may recover the same from the person in possession of the same and the only defense the latter may have is if he has acquired it in good faith at a public sale in which case the owner cannot obtain its return without reimbursing the price paid therefore. DIZON v. SUNTAY G.R. No. L-30817, 29 September 1972 [DIVINO] FACTS: Respondent Lourdes Suntay is the owner of a three-carat diamond ring valued at P5,500.00. On June 13, 1962, respondent and Clarita R. Sison entered into a transaction wherein the respondent's ring was delivered to Sison for sale on commission. Upon receiving the ring, Sison executed and delivered to the respondent a receipt. After the lapse of a considerable time without Sison having returned to the respondent the latter's ring, respondent made demands on Sison for the return of her ring but the latter could not comply with the demands because, without the knowledge of the respondent, on June 15, 1962 or three days after the ring above-mentioned was received by Sison, said ring was pledged by Melia Sison, niece of the husband of Clarita R. Sison, evidently in connivance with the latter, with petitioner Dominador Dizon's pawnshop for P2,600.00. The lower court rendered judgment declaring that respondent Suntay had the right to the possession of the ring in question. The Court of Appeals affirmed said judgement. Hence, the petition for review with the Supreme Court. ISSUE: Can Suntay recover the diamond ring? RULING: YES. The controlling provision is Article 559 of the Civil Code. Respondent Suntay, having been unlawfully deprived of the diamond ring in question, was entitled to recover it from petitioner who was found in possession of the same. The only exception the law allows is when there is acquisition in good faith of the possessor at a public sale, in which case the owner cannot obtain its return without reimbursing the price. The right of the owner cannot be defeated even by proof that there was good faith in the acquisition by the possessor. The common law principle that were one of two innocent persons must suffer by a fraud perpetrated by another, the law imposes the loss upon 75 4S 2014-2015  

San Beda College of Law CIVIL LAW REVIEW   the party who, by his misplaced confidence, has enabled the fraud to be committed, cannot be applied in a case which is covered by an express provision of the new Civil Code, specifically Article 559. Between a common law principle and a statutory provision, the latter must prevail in this jurisdiction. LEDESMA v. COURT OF APPEALS G.R. No. 86051, 1 September 1992 [DOMINGO] FACTS: On September 27, 1977, a person representing himself to be Jojo Consunji, purchased purportedly for his father, a certain Rustico T. Consunji, two brand new motor vehicles(Isuzu Gemini and Holden Premier Vehicle) from Citiwide Motors, Inc. Citiwide Motors delivered the two-above described motor vehicles to the person who represented himself as Jojo Consunji, allegedly the son of the purported buyer, Rustico T. Consunji, and said person in turn issued a manager’s check of the PCIB for the amount of P101,000.00 as full payment of the value of the two motor vehicles.However, when Citiwide Motors deposited the said check, it was dishonored by the bank on the ground that it was tampered with, the correct amount of P101.00 having been raised to P101,000.00 per the bank’s notice of dishonor. Citiwide Motors reported to the Philippine Constabulary the criminal act perpetrated by the person who misrepresented himself as Jojo Consunji and in the course of the investigation, Citiwide Motors learned that the real identity of the wrongdoer/impostor is Armando Suarez who has a long line of criminal cases against him for estafa using his similar modus operandi. On October 17, 1977, plaintiff-appellant was able to recover the Holden Premier vehicle which was found abandoned somewhere in Quezon City. On the other hand, Citiwide Motors learned that the Isuzu Gemini was transferred by Armando Suarez to third persons and was in the possession of one Jaime Ledesma at the time plaintiff-appellant instituted this action for replevin on November 16, 1977. In his defense, Jaime Ledesma claims that he purchased and paid for the subject vehicle in good faith from its registered owner, one Pedro Neyra, as evidenced by the Land Transportation Commission Registration Certificate. After posting the necessary bond in the amount double the value of the subject motor vehicle, plaintiff-appellant was able to recover possession of the 1977 Isuzu Gemini as evidenced by the Sheriff’s Return. ISSUE: Is Citiwide Motors unlawfully deprived of the cars when it sold the same to a person who purported to be authorized by the buyer to get hold of the same on the faith of the manager’s check issued? RULING: YES. A party who has lost any movable or has been unlawfully deprived thereof can recover the same from the present possessor even if the latter acquired it in good faith and has, therefore, title thereto for under the first 76 4S 2014-2015  

San Beda College of Law CIVIL LAW REVIEW   sentence of Article 559, such manner of acquisition is equivalent to a title. There are three requisites to make possession of movable property equivalent to title, namely: (a) the possession should be in good faith; (b) the owner voluntarily parted with the possession of the thing; and (c) the possession is in the concept of owner. Undoubtedly, one who has lost a movable or who has been unlawfully deprived of it cannot be said to have voluntarily parted with the possession thereof. This is the justification for the exceptions found under the second sentence of Article 559 of the Civil Code. There was a perfected unconditional contract of sale between private respondent and the original vendee. The former voluntarily caused the transfer of the certificate of registration of the vehicle in the name of the first vendee—even if the said vendee was represented by someone who used a fictitious name—and likewise voluntarily delivered the cars and the certificate of registration to the vendee’s alleged representative. Title thereto was forthwith transferred to the vendee. The subsequent dishonor of the check because of the alteration merely amounted to a failure of consideration which does not render the contract of sale void, but merely allows the prejudiced party to sue for specific performance or rescission of the contract, and to prosecute the impostor for estafa under Article 315 of the RPC. Thus, the Court ruled that considering that Ledesma had successfully proven that he acquired the property in good faith from the vendor and for a valuable consideration, he was unlawfully divested of the car in his possession.

POSSESSION: Fruits  

LEONARDO AZARCON, MANUEL AZARCON and ESTEBAN ABOBO v. VICTOR EUSEBIO G.R. No. L-11977, 29 April 1959 [ESTEBAN]

FACTS: Victor Eusebio and petitioners herein had a dispute over the possession of a certain parcel of public land. The conflict was ordered to be investigated on May 25, 1955 by the Director of Lands and again on August 3, 1955 by the Secretary of Agriculture and Natural Resources. Before the dispute could be settled, Victor Eusebio filed a complaint in the Court of First Instance of Nueva Ecija praying that defendants be ordered to vacate the six hectares occupied by them and pay damages. Defendant Azarcon answered the complaint alleging that he is in actual possession of a portion of 24 hectares since 1941 by virtue of a homestead application and that the lease application of plaintiff is subsequent to said homestead application of Leonardo Azarcon. He, therefore, prayed that the action be dismissed.

77 4S 2014-2015  

San Beda College of Law CIVIL LAW REVIEW   The answer was filed on June 2, 1954 and on motion of plaintiff dated March 15, 1955, the defendants were declared in default. A motion to set aside the default was denied, and a judgment by default was entered by the court on April 26, 1955. It ordered defendants to restore possession of the land to plaintiff. Having failed to obtain a reconsideration of the above decision, defendants appealed to the Court of Appeals. While the case was pending in the Court of Appeals, a writ for the execution of the judgment of the lower court was issued. Said writ of execution was furnished to the defendants on October 7, 1955. The evidence shows that in spite of the receipt by the defendants of the notice of the writ of execution of October 3, 1955, which writ of execution commanded defendants "to forthwith remove from said premises and that plaintiff have restitution of the same," defendants-appellants nevertheless entered the land to gather palay which was then pending harvest. ISSUE: Whether the defendants’ act of entering the contested land to gather palay which was then pending harvest was lawful despite receipt of order of execution commanding them to vacate said premises. RULING: YES. The Court gathered further from the record that the rice found on the disputed land at the time of the service of the order of execution had been planted by defendants-appellants, who appear to have been in possession of the land from 1951. While the court order of October 3, 1955 ordered the defendant-appellant to move out from the premises, it did not prohibit them from gathering the crop then existing thereon. Under the law a person who is in possession and who is being ordered to leave a parcel of land while products thereon are pending harvest, has the right to a part of the net harvest, as expressly provided by Article 545 of the Civil Code. ART. 545. If at the time the good faith ceases, there should be any natural or industrial fruits, the possessor shall have a right to a part of the expenses of cultivation, and to a part of the net harvest, both in proportion to the time of the possession. As the order of execution did not expressly prohibit the defendants-appellants from gathering the pending fruits, which fruits were the result of their possession and cultivation of the land, it cannot be said that the defendantsappellants committed an act which is clear violation of the courts' order. Besides, the defendants-appellants had presented, after receipt of the order of execution, a motion to set aside the said order of execution, and this motion to stay execution was granted. Defendants furthermore presented a bond in accordance with the order of the court and had it approved by the Court of First Instance. It was perhaps in expectation of this resolution of the court setting aside the order of execution that defendants-appellants may have felt justified in entering the land and harvesting the fruits existing thereon. Again the order of the court setting aside its order to stay execution was issued in the belief that the defendants-appellants had not presented before 78 4S 2014-2015  

San Beda College of Law CIVIL LAW REVIEW   the Court of First Instance of Nueva Ecija and which said court actually approved). Under the circumstances above stated, the Court is not ready to conclude that the defendants-appellants can be held to have committed a clear defiance of the order of the court. Their act in harvesting the pending fruits was not only justified by law but was not expressly prohibited by the court's order, and was even ratified when the court ordered the suspension of the execution. There was, therefore, no open, clear and contumacious refusal to obey a definite order of the court such as would constitute contempt. Furthermore, a person who has been ordered to leave certain premises is ordinarily not prohibited from taking with him his own effects and possession, unless there is an express prohibition to this effect. No such, prohibition was contained in the order for the defendants to leave the land. There may have been a technical violation of an order not to enter the premises, but not of one prohibiting them from removing anything therefrom. Such technical violation of the order cannot be considered as one amounting to a defiance of the court's authority, punishable as contempt. FELIPA CORDERO (Deceased) MAURO OCAMPO, CASIMIRO OCAMPO and ELISEA OCAMPO v. VICTORIA P. CABRAL, ALEJANDRO BERBOSO, DALMACIO MONTAOS and HONORABLE COURT OF APPEALS G.R. No. L-36789, 25 July 1983 [FERNANDEZ] FACTS: Mr. Gregorio Z. Ocampo, husband of Felipa Cordero and father of the other petitiones, died on May 17, 1958, and left several properties. Petitioners took possession of the properties left by him, among others is a riceland. However, they found out that a portion of the same was possessed by Victoria P. Cabral, Alejandro Berboso and Dalmacio Montaos. Petitioners filed a civil case alleging that Victoria P. Cabral continued claiming to be the owner of the land while her co-defendants continued recognizing her as the owner thereof instead of the plaintiffs despite demands to vacate the property. They also claim that due to respondents' occupancy of the aforementioned portion of land, petitioners failed to realize a yearly harvest of at least ten (10) cavanes of palay at the rate of P10.00 per cavan, from the harvest-time of 1958 up to the present. RTC dismissed the complaint. On appeal, even though the CA found that the disputed piece of land is registered in the name of the petitioners but because of the supposed oral sale of the same to the predecessors of the defendants mentioned by the petitioners on appeal, it affirmed the judgment of the trial court dismissing the complaint for the recovery of the land. ISSUES: 1. Are the heirs of the registered owner entitled to the land? 2. May the respondents be held liable for reimbursement of fruits received?

79 4S 2014-2015  

San Beda College of Law CIVIL LAW REVIEW   RULING: 1. YES. The Court of Appeals found as a fact that the disputed portion of the land is admittedly part of the land originally registered in the name of petitioners’ predecessor in interest. There should be no question that that title had become imprescriptible and the original registrant as well as his successors had the right to vindicate their ownership against anybody else. 2. YES. The respondents, by their own admission, are in possession of the disputed land. There is no evidence that they were possessors in bad faith. However, their good faith ceased when they were served with summons to answer the complaint. As possessors in bad faith from the service of the summons they "shall reimburse the fruits received and those which the legitimate possessor could have received”.

POSSESSION: Expenses

  MARTIN MENDOZA and NATALIO ENRIQUEZ v. MANUEL DE GUZMAN G.R. No. L-28721, 5 October 1928 [J. FERNANDO]

FACTS: In a cadastral proceeding, a piece of land was adjudicated in favor of Martin Mendoza and Natalio Enriquez in equal parts pro indiviso subject to the right of retention on the part of Manuel de Guzman until he shall have been indemnified for the improvements existing on the land. Being unable to come to an agreement as to the amount which should be allowed for the improvements made on the land, Martin Mendoza and Natalio Enriquez began an action requesting the court to fix the value of the necessary and useful expenses incurred by Manuel de Guzman in introducing the improvements. The trial court resolved the questions presented by holding that in accordance with the provisions of articles 435 and 454 in relation with article 361 of the Civil Code, the value of the "indemnization" to be paid to the defendant should be fixed according to the necessary and useful expenses incurred by him in introducing "las plantaciones en cuestion." ISSUE: Is the term “indemnizacion” pertains to the amount of expenditures such as necessary and useful expenses? RULING: YES. The amount of the "indemnizacion" is the amount of the expenditures mentioned in articles 453 and 454 of the Civil Code, which in the present case is the amount of the necessary and useful expenditures incurred by the defendant. Necessary expenses have been variously described by the Spanish commentators as those made for the preservation of the thing; as those without which the thing would deteriorate or be lost; as those that augment the income of the things upon which they are expanded. Among the necessary expenditures are those incurred for cultivation, production, upkeep, 80 4S 2014-2015  

San Beda College of Law CIVIL LAW REVIEW   etc.. Here the plaintiffs have chosen to take the improvements introduced on the land and are disposed to pay the amount of the necessary and useful expenses incurred by the defendant. Inasmuch as the retentionist, who is not exactly a posessor in good faith with in the meaning of the law, seeks to be reimbursed for the necessary and useful expenditures, it is only just that he should account to the owners of the estate for any rents, fruits, or crops he has gathered from it. ROBLES AND MARTIN v. LIZZARAGA HERMANOS, et.al. G.R. No. L-16736, 22 December 1921 [L. FERNANDO] FACTS: Anastasia de la Rama died leaving six children, one of whom was Evarista Robles, and some properties, among which was house No. 4 on Iznart Street in Iloilo City. The heirs of Anastasia then entered into partnership with Lizarraga Hermanos (“LH”) in liquidation and settlement of their accounts, by virtue of which the court awarded to said partnership the properties left by the deceased, including the aforesaid house No.4. Evarista, since before the death of her mother Anastasia, has been with her husband occupying house No. 4, at the beginning, by permission of her mother, later on by the consent of her coheirs, and lastly by agreement with the partnership, LH, to whom it had been awarded, paying to said partnership P40 monthly as rent of the upper story. Sometime, LH notified Evarista that the rent would be raised to P60 a month. Evarista refused to pay the new rate and to vacate the house. LH brought suit against her for ejectment while Evarista sued LH to recover the value of the improvements she introduced in house no.4. Evarista claims that LH agreed to sell her the said house, the deed of sale to be executed later; that by virtue of this contract she remained in the occupation of the building and made the improvements; that, as one of the stipulations in the contract of sale of the estate, Evarista assumed the liability of several encumbrances on the estate all of which payments were made through LH. ISSUE: Is Evarista Robles the owner of the aforesaid improvements and has the right to demand payment of their value? RULING: YES. Under Article 453 (now Article 546) of the Civil Code, a possessor in good faith who makes useful improvements on the estate possessed is entitled to demand payment of the value thereof and to retain estate until the expenditures incurred therein are paid to him. Here, the presumption of good faith in favor of Evarista Robles' possession at the time she made the improvements on the property was neither disputed nor discussed, but on the contrary, there is positive evidence sufficient to support the conclusion that when she made the improvements on the aforesaid building she was possessing it in good faith. Moreover, the expenditures incurred in these 81 4S 2014-2015  

San Beda College of Law CIVIL LAW REVIEW   improvements consisting of the addition of a dining room, kitchen, closet, and bathroom in the lower and upper stories of the house, and a stable, suitable as a coach house and dwelling were not necessary inasmuch as without them the house would have continued to stand just as before, but were useful, inasmuch as with them the house better serves the purpose for which it was intended, being used as a residence. If the improvements are useful and Evarista Robles possession was in good faith, the conclusion set out in article 453 of the Civil Code, supra, is inevitable; Evarista Robles is the owner of such improvements, and entitled to reimbursement therefor, and to retain the building until the same is made METROPOLITAN WATERWORKS AND SEWERAGE SYSTEM v. COURT OF APPEALS   G.R. No. L-54526, 25 August 1986 [JAVIER] FACTS: The City of Dagupan filed a complaint against the former NAWASA (now MWSS), for recovery of the ownership and possession of the Dagupan Waterworks System. NAWASA interposed as one of its special defenses R.A. 1383 which vested upon it the ownership, possession and control of all waterworks systems throughout the Philippines and as one of its counterclaims the reimbursement of the expenses it had incurred for necessary and useful improvements amounting to P255,000.00. The trial court found that useful expenses were made in utter bad faith for they were instituted after the complaint was filed and after numerous Supreme Court decisions were promulgated declaring unconstitutional the taking by NAWASA of the patrimonial waterworks systems of cities, municipalities and provinces without just compensation. In support of its claim for removal of said useful improvements, MWSS argues that the pertinent laws on the subject, particularly Articles 546, 547 and 549 of the Civil Code of the Philippines, do not definitely settle the question of whether a possessor in bad faith has the right to remove useful improvements. ISSUE: Does a possessor in bad faith have the right to remove useful improvements? RULING: NO. Article 449 of the Civil Code of the Philippines provides that "he who builds, plants or sows in bad faith on the land of another, loses what is built, planted or sown without right to indemnity." As a builder in bad faith, NAWASA lost whatever useful improvements it had made without right to indemnity

82 4S 2014-2015  

San Beda College of Law CIVIL LAW REVIEW   Moreover, under Article 546 of said code, only a possessor in good faith shall be refunded for useful expenses with the right of retention until reimbursed; and under Article 547 thereof, only a possessor in good faith may remove useful improvements if this can be done without damage to the principal thing and if the person who recovers the possession does not exercise the option of reimbursing the useful expenses. The right given a possessor in bad faith is to remove improvements applies only to improvements for pure luxury or mere pleasure, provided the thing suffers no injury thereby and the lawful possessor does not prefer to retain them by paying the value they have at the time he enters into possession (Article 549, Id.).

 

83 4S 2014-2015  

View more...

Comments

Copyright ©2017 KUPDF Inc.
SUPPORT KUPDF